Tag Archives: ChuyenDe

Góc định hướng và ứng dụng

Góc định hướng. 

Góc giữa hai tia. Cho hai tia $Ox, Oy$, ta cho tia $Ot$ lúc đầu trùng với $Ox$ và cho $Ot$ quay quanh $O$, đến khi $Ot$ trùng với $Oy$, ta nó $Ot$ tạo ra một góc lượng giác (góc định hướng) có tia đầu là $Ox$ tia cuối là $Oy$, kí hiệu $(Ox, Oy)$.

Chú ý: Với hai tia $Ox, Oy$ thì có vô số góc lượng giác có tia đầu $Ox$ tia cuối $Oy$ và hơn kém nhau $k2 \pi$.

Góc giữa hai đường thẳng. Cho hai đường thẳng $a, b$ cắt nhau tại $O$, ta cho đường thằng $t$ qua $O$ lúc đầu trùng với $O$, quay $t$ quanh $O$ đến khi $t$ trùng $a$, ta nói $t$ tạo ra góc giữa đường thẳng $a, b$, kí hiệu là $(a;b)$.

Các góc lượng giác tạo giữa $a, b$ hơn kém nhau $k\pi$.

Một số tính chất thường sử dụng. 

Tính chất 1. Hệ thức Charles
a) Cho $a, b, c$ là ba đường thẳng bất kì thì $(a, b)=(a, c)+(c, b)(\bmod \pi)$
b) $\mathrm{Cho} O x, O y, O z$ là ba tia thì $(O x, O y)=(O x, O z)+(O z, O y)(\bmod 2 \pi)$

Tính chất 2. (Điều kiện 3 điểm thẳng hàng) Cho 3 điểm $A, B, C$ và đường thẳng $d$. Khi đó $A, B, C$ thẳng hàng khi và chỉ khi $(A B, d)=(A C, d)(\bmod \pi)$

Tính chất 3. (Điều kiện 4 điểm đồng viên) Cho 4 điểm $A, B, C, D .$ Khi đó $A, B, C, D$ cùng thuộc một đường tròn khi và chỉ khi $(A C, A D)=(B C, B D)(\bmod \pi)$.

Tính chất 4. Nếu $a$ là phân giác của góc tạo bởi hai đường thẳng $b, c$ thì $(b, a)=-(c, a)=\frac{1}{2}(b, c)\left(\bmod \frac{\pi}{2}\right)$

Tính chất 5. Nếu $a$ và $a^{\prime}$ đỗi xứng nhau qua đường thẳng $d$ thì $(a, d)=-\left(a^{\prime}, d\right)(\bmod \pi)$.

Tính chất 6 . Nếu $a^{\prime}$ là ảnh của $a$ qua phép quay với góc quay $\alpha$ thì $\left(a, a^{\prime}\right)=\alpha(\bmod \pi)$

Các ví dụ

Bài 1. (Định lý Migel) Cho tam giác $A B C$; Gọi $D, E, F$ lần lượt là các điểm thuộc các đường thẳng $B C, A C$ và $A B$
a) Chứng minh rằng các đường tròn ngoại tiếp các tam giác $A E F, B F E, C D E$ cùng đi qua một điểm $M$.
b) Nếu $D, E, F$ thẳng hàng thì điểm $M$ thuộc đường tròn ngoại tiếp của tam giác $A B C$; hơn nữa tâm các đường tròn $(A B C),(A E F),(B F E),(C D E)$ cùng thuộc một đường tròn và đường tròn đó qua $\mathrm{M}$.

Lời giải

a) Gọi $M$ là giao điểm của $(A E F)$ và $(B D F)$, ta chứng minh $C, D, E, M$ đồng viên.
Ta có $(E M ; E C)=(E M ; E A)=(F M ; F A)(\bmod \pi)($ Do $A, E, M, F$ đồng viên $)$
Mà $(F M, F A)=(F M ; F B)=(D M: D B)(\bmod \pi)($ Do $D, M, F, B$ đồng viên $)$
Suyra $(E M ; E C)=(D M ; D B)=(D M ; D C)(\bmod \pi)$
Do đó $M, E, C, D$ đồng viên.

b) $\operatorname{Tacó}(A M: A F)=(E M ; E F)(\bmod \pi),(A M: A F)=(C M ; C B)(\bmod \pi)$ và $(C M ; C B)=(E M ; E D)(\bmod \pi)$
Do đó $E, D, F$ thẳng hàng khi và chỉ khi $(E M ; E F)=(E M: E D)$ khi và chỉ khi $(A M ; A F)=(C M ; C B)$ khi và chỉ khi $A \cdot B, C, M$ đồng viên.
Gọi $O, O_{a}, O_{b}, O_{c}$ lần lượt là tâm đường tròn ngoại tiếp các tam giác $A B C, A E F, B D F, C D E$. Ta chứng minh $O, M, O_{a}, O_{b}, O_{c}$ đồng viên.
Thật vậy ta có $\left(O_{a} M ; O_{a} O_{b}\right)=(E M ; E F)=(C M ; C D)=\left(O M ; O O_{b}\right)(\bmod \pi)$. Do đó $O_{a}, M, O, O_{b}$ đồng viên. Tương tự $O_{a}, M, O, O_{c}$ đồng viên. Suy ra điều cần chứng minh.

Bài 2. (Đường thẳng Steiner – Điểm Antisteiner)

a) Cho tam giác $A B C$ nội tiếp đường tròn $(O), M$ là một điểm thuộc $(O)$. Gọi \$latex $A^{\prime}, B^{\prime}, C^{\prime} \$$ lần lượt là điểm đối xúng của $M$ qua $B C, A C, A B$. Chứng minh rằng $A^{\prime}, B^{\prime}, C^{\prime}$ cùng thuộc một đường thẳng và đường thẳng đó qua trực tâm $\mathrm{H}$ của tam giác $A B C$.
b) Ngược lại lấy $d$ là một đường thẳng qua $H$. Gọi $d_{a}, d_{b}, d_{c}$ lần lưọt là các đường thẳng đối xứng của d qua BC, $A C, A B$. Chúng minh rằng $d_{a}, d_{b}, d_{c}$ đồng qui tai một điểm thuộc đường tròn $(O)$.

Lời giải

a) Gọi $H_{c}, H_{b}$ là điểm đõi xứng của $H$ qua $A B: A C$. Ta có $H_{c}, H_{b} \in(A B C)$
a) $\left(H C^{\prime} ; H B^{\prime}\right)=\left(H C^{\prime} ; H A\right)+\left(H A ; H B^{\prime}\right)=-\left(H_{c} M ; H A\right)-\left(H_{b} A ; H_{b} M\right)=0($ $\bmod \pi)$
Vầy $H, B^{\prime}, C^{\prime}$ thẳng hàng.

b) Ta thấy $H_{a} \in d_{a}, H_{b} \in d_{b} \cdot$ Gọi $M$ là giao điểm của $d_{a}, d_{b}$. Ta chứng minh $M \in(A B C)$. Ta có:
$$
\begin{aligned}
&\left(M H_{a} ; M H_{b}\right)=\left(A^{\prime} H_{a} ; A^{\prime} C\right)+\left(A^{\prime} C ; C A\right)+\left(C A ; M H_{b}\right) \\
&=-\left(A^{\prime} H ; B C\right)+(C B ; C A)-\left(C A ; B^{\prime} H\right) \\
&=\left(B C ; A^{\prime} H\right)+\left(B^{\prime} H ; C A\right)+(C B ; C A) \\
&=2(B C ; C A)(\bmod \pi) \\
&=\left(C H_{a} ; C H_{b}\right)(\bmod \pi)
\end{aligned}
$$
Do đó $M \in(A B C)$.

Bài 3. 
a) Cho tam giác $A B C$ nội tiếp đương tròn $(O), P Q$ là đương kính. Chứng minh rằng đường thẳng Simson của tam giác ABC úng vói các điểm $P, Q$ vuông góc nhau.
b) Tổng quát hơn, nếu $P Q$ là dây cung bất kì thì góc tạo bởi hai đương thẳng Simson ứng với $P$ và $Q$ bằng nủa số đo chung nhỏ $P Q$.

Lời giải

b)
$$
\begin{aligned}
(\mathrm{DI} ; \mathrm{JK}) &=(\mathrm{DI} ; \mathrm{DP})+(\mathrm{DP} ; \mathrm{AC})+(\mathrm{AC} ; \mathrm{CJ})+(\mathrm{CJ} ; \mathrm{JK})(\bmod \pi) \\
&=(\mathrm{CI} ; \mathrm{CP})+1 / 2 \pi+(\mathrm{AC} ; \mathrm{BC})+(\mathrm{QC} ; \mathrm{QK})(\bmod \pi) \\
&=(\mathrm{CB} ; \mathrm{CP})+1 / 2 \pi+(\mathrm{AC} ; \mathrm{BC})+(\mathrm{CQ} ; \mathrm{CK})+(\mathrm{CK} ; \mathrm{QK})(\bmod \pi) \\
&=(\mathrm{CB} ; \mathrm{CP})+(\mathrm{AC} ; \mathrm{CB})+(\mathrm{CQ} ; \mathrm{CA})(\bmod \pi) \\
&=(\mathrm{CQ} ; \mathrm{CP})(\bmod \pi) \square
\end{aligned}
$$

Bài 4. (Chọn đội dự tuyển PTNK 2008) Cho tam giác ABC. Các điểm $M, N, P$ lần luợt thuộc các đt $B C, C A$, AB sao cho tam giác MNP và tam giác $A B C$ đồng dạng. Chúng minh ràng tâm đưòng tròn ngoại tiếp của tam giác $B C$ là thục tâm của tam giác $M N P$.

Lời giải

Theo định lý Migel thì các đường tròn (ANP), (BMP) và (CMN) cắt nhau tại $O$. Ta có
$$
\begin{aligned}
(\overline{O B} ; \overrightarrow{O C}) &=(\overline{O B} ; \overline{O P})+(\overline{O P} ; \overline{O M})+(\overline{O M} ; \overrightarrow{O C}) &(\bmod \pi) \\
&=(\overline{M B} ; \overline{M P})+(\overline{O P} ; \overline{O M})+(\overline{O M} ; \overrightarrow{O C}) &(\bmod \pi) \\
&=-(\overline{M P} ; \overline{M B})+(\overline{B P} ; \overline{B M})+(\overline{N M} ; \overrightarrow{N C}) \quad(\bmod \pi) \\
&=-(\overline{M P} ; \overline{M N})-(\overline{M N} ; \overline{M B})+(\overline{B P} ; \overline{B M})+(\overline{N M} ; \overrightarrow{N C}) \\
&=(\overline{M N} ; \overline{M P})+(\overline{M B} ; \overline{M N})+(\overline{M N} ; \overline{C N})+(\overline{B P} ; \overline{B M}) \\
&=(\overline{M N} ; \overline{M P})+(\overline{M B} ; \overline{C N})+(\overline{B P} ; \overline{B M}) \\
&=(\overline{M N} ; \overline{M P})+(\overline{B P} ; \overline{C N})=2(A B ; A C) \quad(\bmod \pi)
\end{aligned}
$$
Từ đó ta có $\mathrm{O}$ là tâm đường tròn ngoại tiếp tam giác $\mathrm{ABC}$.

Mặt khác
$$
\begin{aligned}
(O N ; P M) &=(O N ; O P)+(O P ; P M) & &(\bmod \pi) \\
&=(A N ; A P)+(B O ; B M) & &(\bmod \pi) \\
&=(A C ; A B)+(B O ; B C) & &(\bmod \pi) \\
&=\frac{\pi}{2} \quad &(\bmod \pi)
\end{aligned}
$$
Suy ra $\mathrm{ON} \perp \mathrm{PM}$. Chứng minh tương tự ta có $\mathrm{MO} \perp \mathrm{NP}$. Hay $\mathrm{O}$ là trực tâm của tam giác $\mathrm{ABC}$.

Bài 5. Cho hai hình vuông $A B C D$ và $A E F G$ cùng hướng, $A, B, E$ không thẳng hàng. Chứng minh rẳng $B E, C F, D G$ đồng quy.

Lời giải

 

Xét phép quay tâm A góc quay $(A B: A D)=90^{\circ}$. Khi đó $B$ biên thành $D, E$ biên thành $G$. Gọi $H$ là giao điểm của $\mathrm{BE}$ và $\mathrm{GD}$. Khi đó $(B E ; G D)=(A B ; A D)=(C B ; C D)=90^{\circ}(\bmod \pi)$. Suy ra $A, H, B, C, D$ đồng viên.
Từ đó ta có $(H B: H C)=(A B: A C)(\bmod \pi)$,
Hơn nữa, $(H G ; H E)=(A G ; A E)=90^{\circ}(\bmod \pi)$ nên $A, E, H, G, F$ cũng đồng viên. Suy ra $(H E ; H F)=(A B: A C)(\bmod \pi)$
Ta có $(H B ; H C)=(H E ; H F)(\bmod \pi)$ mà $H, E, B$ thẳng hàng nên $H, C, F$ thẳng hàng, hay $B E . C F, D G$ đồng quy.

Bài tập rèn luyện

Bài 1 (VMO 2006) Cho tứ giác lồi $A B C D$. Xét một điểm $M$ di động trên đường thẳng $A B$ sao cho $M$ không trùng với $A$ và B. Gọi $N$ là giao điểm thứ hai khác $M$ của đường tròn đi qua 3 điểm $M, N, C$ và đường tròn đi qua 3 điếm $M, B$, D. Chứng minh:
a) Điểm $\mathrm{N}$ di động trên một đường tròn cố định.
b) Đường thẳng MN luôn đi qua một điểm cố định.

Bài 2. Cho tứ giác lồi $A B C D$ nội tiếp một đường tròn. Gọi $P, Q, R, S$ là giao điểm của các đường phân giác ngoài của Các góc ADB và ADB, DAB Và DBA, ACD và ADC, DAC và DCA tương ứng. Chứng minh rẳng $P, Q, R, S$ đồng viên.

Bài 3. Cho tứ giác $A B C$. Chứng minh rằng đường tròn Euler của các tam gíác $A B C, A C D, A B D$ và $B C D$ cùng đi qua một điểm.

Bài 4. Cho hai đường tròn (O) và (O’) cắt nhau tại $A$ và B. Một đường thẳng qua A cắt $(O)$ và $\left(O^{\prime}\right)$ tai $M$ và N. Một đường thẳng qua $B$ cắt $(O)$ và $(O)$ tai $P$ và Q. Chứng minh $M P / / N Q .$

Bài 5. Cho tam giác $A B C$, đưòng cao $A H$ (H thuộc BC). Gọi $D, E$ là hình chiếu của H trên $A B$ và $A D$, đương thẳng $D E$ cắt $B C$ tại $F$. Goi $O_1, O_2$ là tâm đương tròn ngoại tiếp các tam giác BDF và CEF; gọi I là trung điểm $\mathrm{AH}$ và $\mathrm{O}$ là tâm đường tròn ngoại tiếp tam giác $A B C$. Chúng minh rằng 4 điểm $I, O, O_1$ và $O_2$ cùng thuộc một đương tròn.

Ý tưởng chuyển đổi mô hình trong các bài toán hình học phẳng

(Bài viết của Đào Sơn Trà – SV ĐHSP TPHCM)

 

Giới thiệu ý tưởng

Trong tam giác $ABC$ nhọn có $D,E,F$ là các chân đường cao và $H$ là trực tâm. Khi đó:

a) $H$ là tâm đường tròn nội tiếp tam giác $DEF$.
b) $A,B,C$ là tâm bàng tiếp của tam giác $DEF$.

Từ đây ta có thể đổi giữa hai mô hình “bàng tiếp – trực tâm” để xem cách tiếp cận nào thuận lợi hơn để xử lý bài toán. Tất nhiên trong tình huống tam giác tù hoặc vuông cũng có các kết quả tương tự nhưng để đơn giản, ta không đề cập ở đây. Trong các ví dụ, bài tập bên dưới, ta quy ước xét các tam giác nhọn, không cân:

Ví dụ 1.
Cho tam giác $ABC$ có các đường cao $AD,BE,CF$. Gọi $H,K,L$ lần lượt là hình chiếu của $A,B,C$ lên $EF,DF,DE$. Chứng minh rằng các đường thẳng $DH,EK,FL$ đồng quy; các đường thẳng $AH,BK,CL$ đồng quy.

Ta phát biểu lại bài toán như sau: Cho tam giác $DEF$ có $A,B,C$ lần lượt là tâm đường tròn bàng tiếp góc $D,E,F$. Gọi $H,K,L$ lần lượt là hình chiếu của $A,B,C$ lên $EF,DF,DE$. Chứng minh rằng $DH,EK,FL$ đồng quy và $AH,BK,CL$ cũng đồng quy.
Lời giải.

Sau khi chuyển đổi mô hình ta có thể dễ dàng chứng minh được ý a) $DH,EK,FL$ đồng quy (tại điểm Nagel của tam giác $DEF$) bằng cách kết hợp tính chất đường tròn bàng tiếp và định lý Ceva.

Với ý b) ta có: $EF$ là phân giác $\angle DEF$ nên $\angle FEA=\angle DEC$ suy ra $$90^\circ – \angle FEA = 90^\circ – \angle DEC \Rightarrow \angle HAC= \angle LCA$$

Gọi $O$ là giao điểm của $HA$ và $CL$. Khi đó: $$\angle AOC=180^\circ-2\angle HAC=2(90^\circ -\angle HAC)=2\angle BAC$$
nên $AH,CL,BK$ đồng quy tại tâm $(ABC)$.

Ví dụ 2.
Cho tam giác $ABC$ nội tiếp đường tròn $(O;R)$ có $BE,CF$ là hai phân giác cắt nhau tại $I$. $EF$ cắt đường tròn $(O)$ tại hai điểm $M,N$. Chứng minh bán kính đường tròn ngoại tiếp tam giác $IMN$ bằng $2R$.

Ở ví dụ này không xuất hiện trực tiếp yếu tố “trực tâm” hay “tâm bàng tiếp” nhưng ta vẫn có thể vận dụng ý tưởng trên bằng cách xem tâm nội $I$ của tam giác $ABC$ là trực tâm của tam giác tạo bởi $3$ tâm đường tròn bàng tiếp. Cụ thể, ví dụ trên tương đương với bài toán sau:

Cho tam giác $ABC$ nội tiếp $(O;R)$ có đường cao $AD,BE,CF$ cắt nhau tại $H$. Gọi $K,L$ lần lượt là giao điểm của các cặp đường thẳng $(CH,DE),(BH,DF)$. $KL$ cắt đường tròn $Euler$ của tam giác $ABC$ tại $M,N$. Chứng minh bán kính đường tròn ngoại tiếp tam giác $HMN$ bằng $R$.

Lời giải.
Do tứ giác $BDHF$ và $DCEH$ nội tiếp nên ta có:
$$\overline {LD} \cdot \overline {LF} = \overline {LH} \cdot \overline {LB} \Rightarrow P_{L/(DEF)} = P_{L/(BHC)}$$

$$\overline {KC} \cdot \overline {KH} = \overline {KD} \cdot \overline {KE} \Rightarrow P_{K/(DEF)} = P_{K/(BHC)}$$
suy ra $LK$ là trục đẳng phương của $(DEF)$ và $(BHC)$ nên $M,N$ nằm trên $(BHC)$.

Theo tính chất quen thuộc thì $(BHC)$ đối xứng với $(ABC)$ qua $BC$ nên bán kính $(HMN)$ cũng bằng $R$.

Bài tập vận dụng
Bài 1. Cho tam giác $(ABC)$ nội tiếp đường tròn $(O)$. Gọi $M,N,P$ lần lượt là tâm đường tròn bàng tiếp góc $A,B,C$ của tam giác $ABC$. Giả sử $BC$ cắt $NP$ tại $R$ và $T$ là trung điểm cung lớn $BC$ của $(O)$. Chứng minh rằng $MT \bot IR$ với $I$ là tâm đường tròn nội tiếp tam giác $ABC$.

Lời giải
Bài toán được phát biểu lại như sau:

Cho tam giác $MNP$ có $I$ là trực tâm và $A,B,C$ lần lượt là chân đường cao kẻ từ $M,N,P$. $BC$ cắt $NP$ tại $R$. Gọi $T$ là trung điểm cung lớn $BC$ của $(ABC)$. Chứng minh $MI \bot IR$.

Dễ thấy $(ABC)$ là đường tròn $Euler$ của tam giác $MNP$ và $T$ là trung điểm $NP$. Ta sẽ chứng minh $IR$ là trục đẳng phương của $(TM)$ và $(BC)$. \medskip

Ta có:

$$\overline {RA} \cdot \overline {RT} = \overline {RC} \cdot \overline {RB} \Rightarrow P_{R/(NP)} = P_{R/(MT)}$$
$$\overline {IA} \cdot \overline {IM} = \overline {IB} \cdot \overline {IN} \Rightarrow P_{I/(NP)} = P_{I/(MT)}$$

Vậy $IR$ là trục đẳng phương của $(MT)$ và $(NP)$ nên $IR \bot MT$

Bài 2. Cho tam giác $ABC$ nội tiếp đường tròn $(O)$ có phân giác $BE,CF(E \in AC, F \in AB)$. Giả sử $(I)$ là đường tròn nội tiếp tam giác $ABC$. Gọi $M$ là tâm đường tròn bàng tiếp góc $A$. Chứng minh $MO \bot EF$.

Lời giải
Ta phát biểu lại bài toán trên dưới mô hình trực tâm như sau:

Cho tam giác $MNP$ có $I$ là trực tâm và $MA,NB,PC$ là các đường cao. Gọi $NB$ cắt $AC$ tại $E$, $AB$ cắt $PC$ tại $F$. Gọi $O$ là tâm đường tròn $Euler$ của tam giác $MNP$. Chứng minh $MO \bot EF$.

Gọi $O_2$ là tâm ngoại tiếp tam giác $NIP$ thì dễ thấy rằng $O_2$ đối xứng với $O_1$ qua $NP$. Gọi $T$ là trung điểm $NP$ thì $MI = 2O_1T = O_1O_2$. Mà $O_1O_2 \parallel MI$ nên kéo theo tứ giác $MIO_2O_1$ là hình bình hành. Vì thế nên $MO_2$ đi qua trung điểm của $IO_1,$ cũng chính là tâm đường tròn Euler $O$ của tam giác $MNP$.

Tiếp theo, ta thấy rằng

$\overline {EA} \cdot \overline {EC} = \overline {EN} .\overline {EI}$ $\Rightarrow P_{E/(O)} = P_{E/(O_2)}$
$\overline {FA} \cdot \overline {FB} = \overline {FN} \cdot \overline {FI} \Rightarrow P_{F/(O)} =P_{F/(O_2)}$

Suy ra $EF$ là trục đẳng phương của $(O)$ và $(O_2)$ nên $EF \bot OO_2$.

Từ hai điều trên, ta có $EF$ vuông góc với $MO$.

 

Bài 3. Cho tam giác $ABC$ nội tiếp đường tròn $(O)$ và tâm nội tiếp $I$. Đường tròn bàng tiếp $(L)$ của đỉnh $C$ của tam giác $ABC$ tiếp xúc với $AB$ tại $M$. $MI$ cắt $BC$ tại $N$. $P$ là hình chiếu của $C$ lên $LB$. Chứng minh rằng $AI$ và $NP$ cắt nhau trên $(O)$.

Lời giải
Bài toán được phát biểu lại như sau:

Cho tam giác $JKL$ có các đường cao $JA,KB,LC$. Gọi $I$ là trực tâm tam giác $JKL$. Gọi $M$ là hình chiếu của $L$ lên $AB$, $P$ là hình chiếu của $C$ lên $JL$. $MI$ cắt $BC$ tại $N$. Chứng minh rằng $NP$ cắt $JA$ trên đường tròn $Euler$ của tam giác $JKL$.

Gọi $R$ là giao điểm của $JA$ và $NP$. Dễ thấy việc chứng minh $R$ nằm trên đường tròn $Euler$ của tam giác $JKL$ tương đương với việc chứng minh $R$ là trung điểm $IJ$.

Ta có $\Delta LAB \sim \Delta CJB$ mà $LM,CP$ lần lượt là các đường cao nên $\frac{BM}{MA}=\frac{BP}{PJ}$ suy ra $MP \parallel AJ$.

Do $M,I,N$ thẳng hàng nên $P(BI,MN)=B(PI,MN)=B(JK,AC)=-1$ kết hợp với $MP \parallel AJ$ suy ra $R$ là trung điểm $IJ$. Bài toán đã được chứng minh.

Bài 4. Cho tam giác $ABC$ có đường cao $BD,CE$ cắt nhau tại $I$. Chứng minh rằng $AI$ đi qua tâm $Euler$ của tam giác $IDE$.

Lời giải
Dựa vào bổ đề ở \textbf{bài tập 2} ta có thể chuyển bài toán về mô hình sau: \medskip

Cho tam giác $ABC$ có các đường cao $AD,BE,CF$ cắt nhau tại $H$. Gọi $P,Q$ là giao điểm của các cặp đường thẳng $(CH,DE),(BH,DF)$. Lấy $K$ là trực tâm tam giác $HPQ$. Gọi $L$ là tâm $(PKQ)$. Chứng minh $L$ nằm trên $AD$.

Ta có: $$\angle LQP=\frac{180^\circ-\angle QLP}{2}=\frac{180^\circ-2\angle QKP}{2}=90^\circ-\angle QKP=\angle HPK=\angle HCA =\angle LDP$$

Suy ra $QLPD$ nội tiếp. Lại có $LP=LQ$ nên $DL$ là phân giác góc $EDF$ nên $L$ thuộc $AD$. Vậy bài toán đã được chứng minh.

Bài 5.  Chọn đội tuyển 30/4 PTNK 2016 Cho $(O)$ và dây cung $BC$ cố định, điểm $C$ di động. Gọi $I,I_a,I_b$ lần lượt là tâm nội tiếp, tâm bàng tiếp góc $A,B$ của tam giác $ABC$. Gọi $M$ là điểm đối xứng với $I$ qua $O$.

a) Chứng minh rằng $MI_a=MI_b$.
b) Gọi $H,K$ là hình chiếu của $I_b,I_a$ lên $OI$. Đường thẳng qua $H$ vuông góc với $BI_a$ cắt đường thẳng qua $K$ vuông góc với $AI_b$ ở $T$, chứng minh rằng $T$ thuộc đường tròn cố định.

Lời giải
Nhận xét: Khi chuyển đổi sang mô hình trực tâm, giả sử $I_c$ là tâm bàng tiếp góc $C$ của tam giác $ABC$. Ta có $I,O$ lần lượt là trực tâm và tâm đường tròn $Euler$ tam giác $I_aI_bI_c$ nên $M$ là tâm $(I_aI_bI_c)$ từ đó $MI_a=MI_b$. Vậy ta đã giải quyết được ý a) của bài toán.

Ý b) của bài toán sau khi chuyển đổi mô hình, ta có thể dự đoán được $T$ di chuyển trên đường tròn $Euler$ của tam giác $I_aI_bI_c$. Đó là kết quả về cực trực giao của một đường thẳng đi qua tâm ngoại tiếp được phát biểu bởi bài toán sau:

Cho tam giác $ABC$ có đường thẳng $d$ đi qua tâm ngoại tiếp $O$. Gọi $D,E,F$ lần lượt là hình chiếu của $A,B,C$ lên $d$. Chứng minh rằng đường thẳng qua $D,E,F$ vuông góc với $BC,CA,AB$ đồng quy trên đường tròn $Euler$ của tam giác $ABC$.

Gọi $l$ là đường thẳng đi qua trực tâm $H$ của tam giác $ABC$ và vuông góc với $d$. Gọi $S$ là điểm anti-Steiner của $l$. $J$ là điểm đối xứng của $S$ qua $BC$ và $X$ là giao điểm của $SJ$ và $(O)$. $K$ là điểm đối xứng với $H$ qua $BC$.

Ta có: $$\angle AXS =\angle AKS=\angle KHJ$$ suy ra $HJ \parallel AX$.\ Do đó, $D$ nằm trên $AX$ hay $D$ là trung điểm $AX$. Suy ra đường thẳng qua $D$ vuông góc với $BC$ đi qua trung điểm $I$ của $SH$ và nằm trên đường tròn $Euler$ của tam giác $ABC$.

Bài 6. Cho tam giác $ABC$ có phân giác $BE,CF$ cắt nhau tại $I$. Gọi $XP,YQ$ là tiếp tuyến chung ngoài của $(O)$ và $(I_a)$-đường tròn bàng tiếp góc $A$ ($P,Q \in (O)$,$X,Y \in (I_a))$. Chứng minh $P,Q,E,F$ thẳng hàng.

Lời giải
Gọi $I_b,I_c$ là tâm đường tròn bàng tiếp góc $B,C$ để chuyển về mô hình trực tâm thì theo ví dụ I.2 ta cần chứng minh $I,I_c,I_b,P,Q$ cùng nằm trên một đường tròn.

Gọi $M$ là giao điểm của $I_aP$ với $(ABC)$, $K$ là hình chiếu của $O$ lên $XI_a$. \medskip

Theo hệ thức $Euler$ ta có: $$OI_a^2=R^2+2Rr_a$$
suy ra $$PX^2=OK^2=OI_a^2-KI_a^2=R^2+2Rr_a-(r_a-R)^2=4Rr_a-r_a^2$$
ta thu được $PI_a^2=4Rr_a$. Mà $I_aP\cdot I_aM=BI_a^2=OI_a^2-R^2=2Rr_a$. Suy ra $M$ là trung điểm $PI_a$.

Do $(O),I$ là đường tròn $Euler$ và trực tâm của tam giác $I_aI_bI_c$ nên theo Bài tập 2 ta có: ${V_{{I_a}}}^2:(O) \to (I{I_b}{I_c});M \to P$ mà $M \in (O)$ nên $P \in (II_bI_c)$.

Tương tự thì $Q \in (II_bI_c)$ nên ta có được điều phải chứng minh.

Bài 7. Cho tam giác $ABC$ nội tiếp đường tròn $(O)$ và ngoại tiếp đường tròn $(I)$. Gọi $M,N$ là điểm chính giữa cung $BC$ và cung $BAC$ của $(O)$. $NI$ cắt $(O)$ lần thứ hai tại $P$. $MP$ cắt trung trực $AI$ tại $T$. Gọi $S$ là giao điểm tiếp tuyến tại $A$ của $(O)$ với $BC$. Chứng minh rằng $TS \parallel AI$.

Lời giải

Gọi $I_a,I_b,I_c$ lần lượt là tâm đường tròn bàng tiếp góc $A,B,C$ của tam giác $ABC$ để chuyển về mô hình trực tâm. Gọi $X$ là giao điểm của $BC$ và $I_bI_c$. $J$ là giao điểm của $(I_aBIC)$ với $(I_aI_bI_c)$ thì ta có $N,I,J$ thẳng hàng.

Tứ giác toàn phần $BCI_bI_cI_aX$ nội tiếp nên $J$ là điểm Miquel và $I_a,J,X$ thẳng hàng mà $\angle IJI_a =90^\circ$ suy ra tứ giác $AIJX$ nội tiếp.

Ta có: $$\angle MPJ =\angle I_aJI = 90^\circ$$ suy ra $MP \parallel I_aJ$. Lại có $M$ là trung điểm $JI_a$ nên $P$ là trung điểm $IJ$. Suy ra $T$ là tâm $(AIJX)$. Ta thu được $TX=TA$.

Mà $S$ là tâm $A-Apollonius$ của tam giác $ABC$ nên $SX=SA$. Vậy $ST$ là trung trực của $XA$ nên $ST \bot XA$ suy ra $ST \parallel AI$.

Bài 8. (Trích VN TST 2019) Cho tam giác $ABC$ ngoại tiếp $(O)$ và nội tiếp $(I)$. Gọi $E,F$ là giao điểm của các cặp đường thẳng $(BI,AC),(CI,AB)$. Gọi $P,Q$ lần lượt là trung điểm cung $ABC$ và $ACB$. $PQ$ cắt $BC,EF$ tại $G$ và $H$. $EF$ cắt $BC$ ở $K$. Chứng minh rằng tiếp tuyến ứng với $G$ của tam giác $GHK$ vuông góc với $OI$.

Lời giải
Đây là một bài toán hay và khó. Nếu không có cách tiếp cận chuyển đổi mô hình thích hợp thì việc xử lý các tính chất sẽ gặp nhiều khó khăn. Vận dụng ý tưởng ở Ví dụ I.2 ta chuyển bài toán về mô hình trực tâm như sau:

Cho tam giác $ABC$ có các đường cao $AD,BE,CF$. Gọi $M,N,P$ lần lượt là trung điểm của $BC,CA,AB$. Gọi $BH,CH$ cắt $FD,ED$ lần lượt tại $S,T$. $ST$ cắt $PN$ tại $Y$ và cắt $EF$ tại $Z$. Gọi $X$ là giao điểm của $PN$ và $EF$, $K$ là trung điểm $YZ$. Chứng minh rằng: $XK$ vuông góc với đường thẳng $Euler$ của tam giác $ABC$.

Ta có: $$\overline {XN} .\overline {XP} = \overline {XE} .\overline {XF} \Rightarrow P_{X/(APN)} = {{\mathscr{P}}_{X/\left(AEF \right)}} $$
suy ra $AX$ là trục đẳng phương của $(APN)$ và $(AEF)$ nên $AX \bot OH$.

Gọi $U$ là tâm $Euler$ của tam giác $ABC$ thì theo \textbf{Bài tập 2} ta có $AU \bot ST$.

 

Qua $A$ kẻ đường thẳng song song với $YZ$ cắt $EF$ tại $I$ và cắt $PN$ tại $J$ thì $AU \bot IJ$, áp dụng định lý con bướm cho tứ giác $FPEN$ nội tiếp ta thu được $AJ=AI$. Từ đó suy ra $AX$ đi qua trung điểm $YZ$ dẫn đến $A,X,K$ thẳng hàng nên $XK$ vuông góc với $OH$. Vậy ta thu được điều phải chứng minh.

Bài 9. (Trích VN TST 2016) Cho tam giác $ABC$ nội tiếp đường tròn $(O)$ có $B,C$ cố định, $A$ di động trên cung $BC$ của $(O)$. Các phân giác $BE,CF$ cắt nhau tại $I$. $BE,CF$ cắt đường tròn $(O)$ tại $K,L$. $AI$ cắt $KL$ tại $P$. Gọi $Q$ là một điểm trên $EF$ sao cho $QP=QI$. $J$ nằm trên $(BIC)$ sao cho $IJ \bot IQ$. Chứng minh rằng trung điểm $IJ$ di chuyển trên một đường tròn cố định.

Lời giải
Tiếp tục với ý tưởng Ví dụ I.2 Ta dựng $I_b,I_c$ lần lượt là tâm đường tròn bàng tiếp góc $B,C$ của tam giác $ABC$ để chuyển về mô hình trực tâm thì ta thu được $L,K$ lần lượt là trung điểm của $II_c$ và $II_b$.

Gọi $R,S$ là giao điểm của $EF$ với $(O)$(như hình vẽ). $RI,SI$ cắt đường tròn $(O)$ lần thứ hai tại $T,W$. $TW$ cắt $BI$ tại $G$. Đường thẳng qua $I$ vuông góc với $OI$ cắt $LK,BC,SR,TW$ tại $V,U,Q’,X$.

Theo ví dụ 2,ta có $S,R \in (II_bI_c)$. Do đó: $$\angle GTR= \angle ISR=\angle II_bR$$
suy ra tứ giác $GTI_bR$ nội tiếp. Ta thu được $$IG\cdot II_b=IT \cdot IR=IB \cdot IK=\frac{1}{2}IB \cdot II_b$$
suy ra $TW$ đi qua trung điểm $IB$. Tương tự: $TW$ cũng đi qua trung điểm $IC$ nên $TW$ là đường trung bình của tam giác $IBC$.

 

Áp dụng định lý con bướm cho hai dây cung $LC,BK$ cắt nhau tại $I$, ta được $IV=IU$. Tiếp tục áp dụng định lý con bướm cho hai dây cung $SW,TR$, ta được $IX=IQ’$.

Mà $X$ là trung điểm $IU$ nên $Q’$ là trung điểm $IV$ do đó $IQ’=Q’V=Q’P$ suy ra $Q \equiv Q’$. Vậy $OI \bot IQ$. Gọi $O_1$ là trung điểm cung $BC$ không chứa $A$ thì $O_1$ là tâm $(BIC)$. Gọi $M$ là trung điểm $IJ$ khi đó ta có $\angle OMO_1 =90^\circ$ nên $M$ nằm trên $(OO_1)$, là đường tròn cố định. Ta có điều phải chứng minh.

Bài tập tự luyện

  1. Cho tam giác $ABC$ có các đường cao $AD,BE,CF$. Gọi $M,N,P$ là trung điểm của $EF,FD,DE$ và $K$ là tâm nội tiếp tam giác $MNP.$ Gọi $x,y,z$ lần lượt là khoảng cách từ $A\to EF,B\to DF,C\to DE.$ Chứng minh rằng
    $${{x}^{2}}-K{{A}^{2}}={{y}^{2}}-K{{B}^{2}}={{z}^{2}}-K{{C}^{2}}.$$

  2. Cho tam giác $ABC$ có $T$ là trung điểm $BC$ và $X,Y$ là tâm bàng tiếp góc $B,C$ của tam giác $ABC.$ Giả sử $TX$ cắt $AB,AC$ lần lượt tại $M,N,$ còn $TY$ cắt $AB,AC$ lần lượt tại $P,Q.$ Chứng minh rằng $M,N,P,Q$ là các đỉnh của một hình thang ngoại tiếp được đường tròn.

  3. Cho tam giác $ABC$ nội tiếp $(O)$ có tâm nội tiếp $I,$ tâm bàng tiếp góc $A$ là $J.$ Trên các đường thẳng $JB,JC$ lần lượt lấy $M,N$ sao cho $MA=MJ$ và $NA=NJ.$ Đường thẳng $MN$ cắt $IB,IC$ ở $E,F.$ Chứng minh rằng trung tuyến đỉnh $I$ của tam giác $IEF$ chia đôi cung $BAC$ của $(O)$.

  4. Cho tam giác $ABC$ có trực tâm $H$. Đường tròn $(BHC)$ cắt đường tròn Euler của tam giác $ABC$ ở $M,N$. Chứng minh rằng $AM=AN.$

  5. (Bài toán về điểm Bevan) Cho tam giác $ABC$ có $I_a,I_b,I_c$ lần lượt là tâm đường tròn bàng tiếp góc $A,B,C.$ Khi đó, ký hiệu $X$ là tâm đường tròn ngoại tiếp tam giác $I_aI_bI_c,$ cũng chính là điểm Bevan của tam giác $ABC$. Gọi $O,I,G,H$ lần lượt là tâm ngoại tiếp, tâm nội tiếp, trọng tâm, trực tâm của tam giác $ABC.$ Chứng minh rằng $O$ là trung điểm của $XI$ và $G$ là trọng tâm của $HIX.$

GIỚI THIỆU MỘT SỐ SÁCH TOÁN DÀNH CHO CÁC BẠN LỚP 10 CHUYÊN TOÁN

Vào lớp 10 nhiều bạn hỏi tài liệu toán để tự học, sẵn đây mình cũng muốn giới thiệu một số sách cho các em tự học, sách có bản quyền hoặc sách lậu cũng được giới thiệu nha.

Đại số

  •  Bất đẳng thức: Các sách của thầy Võ Quốc Bá Cẩn, google hoặc Tiki chắc là có hết, bán ở nhà sách Hồng Ân – số 20 Nguyễn Thị Minh Khai, Q1, TPHCM.
  • Đa thức: Chuyên Khảo Đa Thức (Lê Hoành Phò – thầy của thầy Trần Nam Dũng), Chuyên đề đa thức (Nguyễn Tài Chung và các tg khác)
  • Phương Trình Hàm (Bài toán Hàm qua các kì thi Olympic – thầy Nguyễn Trọng Tuấn, 2004), sách này sau có của thầy Nguyễn Tài Chung viết lại.

Hình học

  • Tài liệu giáo khoa chuyên toán hình học
  • Các sách của thầy Nguyễn Văn Linh: Một số chủ đề hình học phẳng, 108 bài toán hình học phẳng.
  • website: Hình học – Toán Việt (geosiro.com)

Số học

  •  Number Theory: structures, examples, problems (Sách này có file pdf để gửi sau) của Titu và người khác.
  • Sách của tác giả Đàm Văn Nhỉ, quên tựa đề.

Tổ hợp

  • Problems Solving Strategies – Engel Download
  • 102 bài toán tổ hợp của Titu Download
  • Một số chuyên đề tổ hợp – Tác giả Phạm Văn Phương (ĐHSP HN)
  • Combinatorics Problems and Method  Download

….

Đề thi

  •  mathlinks.ro
  • Đề thi chọn đội dự tuyển và đội tuyển trường Phổ Thông Năng Khiếu 2008 – 2021 (Lê Phúc Lữ và đồng bọn :D)

 

Phương châm

  • Học thầy không tày học bạn, học bạn chưa chắc bằng tự học.
  • Học ít biết nhiều có khi lại tốt hơn học nhiều biết ít.

Chúc các em học tốt!

Hệ thức lượng trong tam giác vuông – Chứng minh đẳng thức P2

Bài 1. Cho tam giác $ABC$ vuông tại $A$ có $BC = 3\sqrt{5}$, hình vuông $ADEF$ có $D$ thuộc $AB$, $E$ thuộc $BC$ và $F$ thuộc $AC$. Biết hình vuông có cạnh 2, tính độ dài các cạnh $AB, AC$ (giả sử $AB < AC$).
Lời giải. Đặt $BD = x, CF = y$, vì $AB < AC$ nên $x < y$.
Ta có $\triangle BDE \backsim \triangle EFC \Rightarrow BD \cdot CF = ED \cdot EF = 4$.
Mặt khác $AB^2 + AC^2 = BC^2 \Rightarrow (x+2)^2+(y+2)^2 = 45 \Rightarrow (x+y)^2 + 4(x+y) -45 = 0 \Rightarrow x+y = 5$.
Suy ra $x(5-x) = 4$, giải ra được $x = 1, y = 4$.
Từ đó suy ra $AB = 3, AC = 6$.

Bài 2. Cho tam giác $ABC$ nhọn trung tuyến $AM$. \begin{enumerate}
a) Chứng minh rằng $4AM^2 + BC^2=2(AB^2+AC^2)$.
b) Vẽ trung tuyến $BN$. Tìm điều kiện về độ dài các cạnh của tam giác $ABC$ để $AM \bot AN$.
Lời giải.
a) Gọi $H$ là chân đường cao kẻ từ $A$, giả sử $H$ nằm giữa $B$ và $M$. Ta có:

$AB^2 + AC^2 = 2AH^2 + BH^2 + CH^2$
$= 2AH^2 + (BM – HM)^2 + (CM + HM)^2 $
$= 2AH^2 + 2HM^2 + 2BM^2 = 2AM^2 + \dfrac{BC^2}{2}$

b) Gọi $G$ là trọng tâm tam giác: $GM=\dfrac{1}{3}AM,GB=\dfrac{2}{3}BN$. Ta có $AM\perp BN$ khi và chỉ khi:\

$GM^2 + GB^2 = BM^2$
$\Leftrightarrow \dfrac{1}{9}AM^2 + \dfrac{4}{9}BN^2 = \dfrac{1}{4}BC^2$
$\Leftrightarrow \dfrac{1}{9} \left( \dfrac{AB^2 + AC^2}{2} – \dfrac{BC^2}{4} \right) + \dfrac{4}{9}\left(\dfrac{AB^2 + BC^2}{2} – \dfrac{AC^2}{4}\right) = \dfrac{BC^2}{4}$
$\Leftrightarrow 5AB^2 = AC^2 + BC^2$

Bài 3. Cho tam giác $ABC$, hai đường phân giác $BD$ và $CE$ cắt nhau tại $I$ thỏa mãn $BD\cdot CE = 2\cdot BI\cdot CI$. Tam giác $ABC$ là tam giác gì? vì sao?
Lời giải.

Đặt $ BC = a, CA = b, AB = c $. Ta có, $ AI $ là phân giác trong $ \triangle ABD $\
Suy ra:
$ \dfrac{BI}{c} = \dfrac{DI}{AD} = \dfrac{BD}{c + AD} \Rightarrow \dfrac{BI}{BD} = \dfrac{c}{c+ AD} $
Chứng minh tương tự
$ \dfrac{CD}{CE} = \dfrac{b}{b + AE} $
Như vậy điều cần chứng minh tương đương với

$\dfrac{BI}{BD} \cdot \dfrac{CI}{CE} = \dfrac{1}{2} \Leftrightarrow \dfrac{bc}{(c + AD)(b + AE)} = \dfrac{1}{2}$
$\Leftrightarrow bc = AD\cdot b + AE\cdot c + AD\cdot AE \qquad (*)$

Mặt khác, trong tam giác $ ABC $ ta có
$ BD $ là phân giác $ \angle ABC$ ta có $\dfrac{AD}{c} = \dfrac{CD}{a} = \dfrac{b}{a + c} \Rightarrow AD = \dfrac{bc}{a + c}$
$ CD $ là phân giác $ \angle ACB$ ta có \dfrac{AE}{b} = \dfrac{BE}{a} = \dfrac{c}{a + b} \Rightarrow AE = \dfrac{bc}{a + b}$
Do đó (*) tương đương với

$bc = \dfrac{b^2c}{a + c} + \dfrac{bc^2}{a + b} + \dfrac{b^2c^2}{(a+b)(a+c)}$
$\Leftrightarrow a^2 = b^2 + c^2$

Vậy tam giác $ ABC$ vuông tại $ A $.

Bài 4. Cho tam giác $ABC$ đều cạnh $a$. $M$ là một điểm thay đổi bên trong tam giác. Gọi $D, E, F$ lần lượt là hình chiếu vuông góc của $M$ trên các cạnh $BC, AC, AB$. Tìm giá trị nhỏ nhất của biểu thức: $P = AF^2 + BD^2 + CE^2$.
Lời giải.
Ta có $AM^2 = AF^2 + MF^2 = AE^2 + ME^2$. Suy ra $AF^2 – AE^2 = ME^2 – MF^2$.
Tương tự $BD^2 – BF^2 = MF^2 – MD^2, CE^2 – CD^2 = MD^2 -MD^2$.
Khi đó $AF^2 -AE^2 + BD^2 – BF^2 + CE^2-CE^2 = 0 \Leftrightarrow AF^2 +DB^2 + CE^2 = AE^2+BF^2+CE^2$.
Mặt khác $AF^2 + BF^2 \geq \dfrac{(AF+BF)^2}{2} = \dfrac{a^2}{4}$.
Tương tự thì $BD^2 +CD^2 \geq \dfrac{a^2}{2}$ và $CE^2+AE^2 \geq \dfrac{a^2}{2}$.
Do đó $P =AF^2 + BD^2 + CE^2 \geq \dfrac{3a^2}{4}$.
Đẳng thức xảy ra khi $D, E, F$ lần là trung điểm của $BC, AC, AB$.
Vậy $P_{min} = \dfrac{3a^2}{4}$.

Bài 5. Cho hình vuông $ABCD$ cạnh $a$. Các điểm $M, N$ lần lượt thay đổi trên cạnh $BC, CD$ sao cho $\angle MAN = 45^\circ$. Chứng minh chu vi tam giác $CMN$ không đổi và tìm giá trị lớn nhất của diện tích tam giác $CMN$.

Trên tia đối của tia $DC$ lấy điểm $K$ sao cho $\angle KAN = \angle MAN = 45^\circ$.
Do $\angle KAD+\angle DAN =45^\circ \quad \text{và} \quad \angle DAN+\angle MAB =45^\circ \quad \text{nên} \quad \angle KAD =\angle MAB$
$\Rightarrow \triangle KAD =\triangle MBA$(ch-cgv) $\Rightarrow AK=AM \quad \text{và} \quad KD=BM$
Khi đó $\triangle KAN=\triangle MAN$(c-g-c) $\Rightarrow MN=KN$
Ta có:
$P_{\triangle CMN}=MN+MC+NC=KN+MC+NC
=KD+DN+NC+MC=BM+MC+NC+ND=DC+CB=2a$.
Vậy chu vi của $\triangle CMN$ luôn không đổi và bằng $2a$
Đặt $MC=x,NC=y$
$P_{\triangle CMN}=MN+MC+NC=x+y+\sqrt{x^2+y^2}=2a$
Áp dụng bất đẳng thức Cauchy:
$2a=x+y+\sqrt{x^2+y^2}\ge 2\sqrt{xy}+ \sqrt{2xy}=\left(\sqrt{2}+2\right)\sqrt{xy} \Rightarrow xy\le \dfrac{4a^2}{(\sqrt{2}+2)^2}$
$S_{\triangle CMN}=\dfrac{1}{2}xy\le \dfrac{1}{2}.\dfrac{4a^2}{6+4\sqrt{2}}=\dfrac{a^2}{\sqrt{2}+3}$

Bài 6. Cho $\triangle A B C$ vuông ờ $A, A H \perp B C, H \in B C . H E \perp A C$,
$H F \perp A B$
\begin{enumerate}
a) Chứng minh rằng $H A^{3}=B F \cdot C E \cdot B C$.
b) Chứng minh rằng $\sqrt[3]{B F^{2}}+\sqrt[3]{C E^{2}}=\sqrt[3]{B C^{2}}$.
c) Gọi $M, N$ là hình chiếu của $E, F$ lên $B C$.
Chứng minh rằng $\sqrt{M C}+\sqrt{N B}=\sqrt{B C}$.
d) Chứng minh rằng $\sqrt[3]{N B \cdot N F}+\sqrt[3]{M C \cdot M E}=\sqrt[3]{A B \cdot A C}$.

Bài 7. Cho tam giác $ABC$ vuông tại $A$, $M$ là điểm thuộc cạnh $BC$ thỏa $MA^2 = MB \cdot MC$. Chứng minh rằng $M$ là trung điểm của $BC$ hoặc $M$ là chân đường cao từ $A$ đến $BC$.

Số hữu tỉ – Vô tỉ

Số hữu tỉ – Số vô tỉ

(Bài viết dành cho các em trung học cơ sở)

Trong bài viết nhỏ này tôi xin giới thiệu một số bài toán liên quan đến các tập hợp số hữu tỉ và vô tỉ, một số trong đó đã xuất hiện trong các kì thi tuyển sinh vào 10 hay các kì thi học sinh giỏi.
Đầu tiên ta xem lại một số khái niệm và tính chất quan trọng.

Định nghĩa. Tập hợp các số có dạng $\dfrac{p}{q}$ trong đó $p, q$ là các số nguyên, $q \neq 0$ được gọi là số hữu tỉ. Kí hiệu là $\mathbb{Q}$. Tập số nguyên là tập con của tập các số hữu tỉ.
Tập hợp các số không phải là số vô tỉ được gọi là số vô tỉ, kí hiệu là $I$.

Tính chất 1. Ta có một số tính chất sau của số vô tỉ và hữu tỉ.

  • Tổng hiệu tích thương của hai số hữu tỉ là hữu tỉ.
  • Tổng, tích, thương của một số hữu tỉ và vô tỉ là một số vô tỉ

Việc chứng minh một số là số hữu tỉ hay vô tỉ chủ yếu dựa vào các định nghĩa trên, trong đó việc chứng minh một số là số vô tỉ hầu hết là sử dụng phương pháp chứng minh phản chứng.
Ta bắt đầu với bài toán cơ bản sau:
Ví dụ 1.
a) Chứng minh $\sqrt{2}$ là một số vô tỉ.
b) Chứng minh $\sqrt{2}+\sqrt{3}$ là một số vô tỉ.

Lời giải.

Ta sử dụng phương pháp chứng minh là phản chứng.

a) Giả sử $\sqrt{2}$ là số hữu tỉ, tức là tồn tại $\dfrac{p}{q}$ trong đó $p, q \in \mathbb{Z},(p,q) = 1, q \neq 0$ và $\sqrt{2}=\dfrac{p}{q}$.
Khi đó ta có $p^2 = 2q^2$, suy ra $p^2$ chia hết cho $2$ mà $2$ nguyên tố nên $p$ chia hết cho $2$, $p = 2k$.
Suy ra $q^2 = 2k^2$, lí luận tương tự thì $q$ chia hết cho $2$, do đó $(p, q) \neq 1$ (mâu thuẫn).
Vậy điều giả sử sai, $\sqrt{2}$ là số vô tỉ.
b) Giả sử $\sqrt{2}+\sqrt{3} = a$ hữu tỉ, suy ra $\sqrt{6} = \dfrac{a^2-5}{2}$ hữu tỉ. Chứng minh tương tự trên ta cũng suy ra điều vô lí.

Từ bài toán trên ta có thể chứng minh bài toán tổng quát sau:

Ví dụ 2. Cho $n$ là số tự nhiên nếu $\sqrt{n}$ không là số tự nhiên thì $\sqrt{n}$ là số vô tỉ.

Lời giải.

Giả sử $\sqrt{n}$ không phải vô tỉ và không phải số nguyên, suy ra $\sqrt{n} = \dfrac{p}{q}$ trong đó $(p,q) =1, q > 1$.
Tương tự ta có $p^2 = nq^2$. Do $q > 1$ nên có ước nguyên tố, giả sử $r$ là một ước nguyên tố của $q$, suy ra $p^2$ chia hết cho $r$, suy ra $p$ chia hết cho $r$, khi đó $(p,q) \neq 1$ (vô lí).
Vậy căn của một số nguyên là một số nguyên hoặc là một số vô tỉ.
\

Đặt $\sqrt{2} = x$, ta có $x^2 = 2 \Leftrightarrow x^2 – 2 = 0$, đến đây ta thấy $\sqrt{2}$ là một nghiệm của phương trình $x^2-2 = 0$. Ta có thể chứng minh phương trình $x^2 -2=0$ không có nghiệm hữu tỉ, từ đó suy ra $\sqrt{2}$ không là số hữu tỉ. Tất nhiên việc chứng minh này không khác mấy chứng minh trên. Tuy nhiên với các nhìn khác, ta có bài toán sau:

Ví dụ 3. Cho phương trình với các hệ số nguyên $a_0, a_1, \cdots, a_n$: $$a_nx^n + a_{n-1}x^{n-1}+\cdots+a_1x + a_0 = 0$$
Khi đó nếu $\dfrac{p}{q}$ với $(p,q)=1$ là một nghiệm hữu tỉ của phương trình thì $p|a_0, q|a_n$.Đặt biệt nếu $a_n=1$ thì nếu phương trình có nghiệm hữu tỉ thì nghiệm là số nguyên.

Lời giải

Thế $\dfrac{p}{q}$ vào phương trình và qui đồng, ta có $$a_np^n+a_{n-1}qp^{n-1}+\cdots+a_1q^{n-1}p + a_0q^n = 0$$
Khi đó $a_np^n$ chia hết cho $q$, suy ra $a_n$ chia hết cho $q$, tương tự thì $a_0$ chia hết cho $p$.

Cũng tương tự, ta có bài toán sau:
Ví dụ 4. Cho phương trình $ax^2 + bx + c = 0$, trong đó $a, b, c$ là các số tự nhiên lẻ. Chứng minh rằng phương trình không có nghiệm hữu tỉ.
Lời giải.

Giả sử $\dfrac{p}{q}, (p,q)=1$ là một nghiệm hữu tỉ của phương trình trên. Khi đó ta có $p|c, q|a$, suy ra $p, q$ đều lẻ. Mặt khác ta có $ap^2 + bpq+ cq^2 = 0$. Vế trái là một số lẻ nên vô lí. Vậy phương trình không có nghiệm hữu tỉ.

Sử dụng bài toàn 3 ta có thể chứng minh $\sqrt{2} + \sqrt{6}$ là số vô tỉ theo một các khác. Bằng cách chứng minh $a = \sqrt{2}+\sqrt{6}$ là nghiệm của phương trình bậc 4: $x^4 – 10x^2 – 1 = 0$, và dễ thấy phương trình trên không có nghiệm hữu tỉ nên $\sqrt{2}+\sqrt{6}$ là số vô tỉ.

Sau đây ta đi tới một số bài toán khác cũng liên quan đến số hữu tỉ và vô tỉ.
Ví dụ 5. Cho các số thực $x, y, z$ khác 0 thỏa $xy, yz, xz$ là các số hữu tỉ.
a) Chứng minh $x^2 + y^2 + z^2 $ là số hữu tỉ.
b) Giả sử $x^3+y^3+z^3$ cũng là số hữu tỉ. Chứng minh $x, y, z$ là các số hữu tỉ.
Lời giải.

a) Ta có $xy, yz \in \mathbb{Q}$, suy ra $\dfrac{x}{z} \in \mathbb{Q}$.
Mà $xz \in \mathbb{Q}$ suy ra $x^2 \in \mathbb{Q}$.
Tương tự ta cũng có $y^2, z^2 \in \mathbb{Q}$.
b) Ta có $x(x^3+y^3+z^3) = (x^2)^2 + (xy)y^2 + (xz)z^2 \in \mathbb{Q}$. Suy ra $x \in \mathbb{Q}$.
Tương tự ta cũng có $y, z \in \mathbb{Q}$.

Chú ý. Với cách giải trên ta chấp nhận không thể xảy ra $x^3+y^3+z^3 = 0$ vì phương trình này không có nghiệm nguyên hay nghiệm hữu tỷ.

Ví dụ 6. Tìm tất cả các số tự nhiên $a, b$ sao cho $$\dfrac{\sqrt{2}+\sqrt{a}}{\sqrt{3}+\sqrt{b}}
$$ là số hữu tỉ.
Lời giải.
Đặt $x = \dfrac{\sqrt{2}+\sqrt{a}}{\sqrt{3}+\sqrt{b}}$ là số nguyên.
Suy ra $\sqrt{a} – x\sqrt{b} = x\sqrt{3}-\sqrt{2}$
Bình phương hai vế ta có $a +x^2b -2x\sqrt{ab} = 3x^2+2-2x\sqrt{6} \Rightarrow a+x^2b-3x^2-2 = 2x(\sqrt{ab}-\sqrt{6})$.
Suy ra $\sqrt{ab}-\sqrt{6} = y \in \mathbb{Q}$.
Khi đó $ab = 6+y^2 – 2y\sqrt{6}$.
Vì $\sqrt{6}$ là số vô tỉ nên đẳng thức xảy ra khi và chỉ khi $y = 0$ và $ab=6$.
Ta xét các trường hợp sau:

  • $a = 1, b = 6 \Rightarrow x = \dfrac{1}{\sqrt{6}}$ vô tỉ.
  • $a = 2, b = 3 \Rightarrow x= \dfrac{\sqrt{2}}{\sqrt{3}}$.
  • $a = 3, b = 2 \Rightarrow x = 1$.
  • $a = 6, b = 1 \Rightarrow x = \sqrt{2}$ vô tỉ.

Vậy $a = 3, b = 2$ là số cần tìm.

Ví dụ 7. Tìm tất cả các bộ số hữu tỉ dương $(x, y, z)$ sao cho $x+\dfrac{1}{y}, y + \dfrac{1}{z}, z+\dfrac{1}{x}$ là các số nguyên.

Lời giải.
Đặt $a = x+\dfrac{1}{y} (1), b = y + \dfrac{1}{z} (2), c = z+\dfrac{1}{x} (3)$.
Từ (1) ta có $y = \dfrac{1}{a-x}, z = \dfrac{1}{b-y} = \dfrac{a-x}{ab-1-bx}$. Thế vào (3) ta có:
$\dfrac{a-x}{ab-1-bx}+\dfrac{1}{x} = c \Leftrightarrow (bc-1)x + (a-b+c-abc)x + ab – 1 = 0$ (4).
Nếu $bc = 1$ thì $b = 1, c = 1$ suy ra $a = 1$. Khi đó $3 = x + \dfrac{1}{x} + y +\dfrac{1}{y} + z + \dfrac{1}{z} \geq 6$ (vô lý)
Nếu $bc \neq 1$, khi đó ta xem (4) như phương trình bậc hai có nghiệm hữu tỷ $x$, khi đó $\Delta = (a-b+c-abc)^2 – 4(bc-1)(ab-1) = (abc-a-b-c)^2 – 4$ là số chính phương.
Đặt $t = abc-a-b-c$ ta có $t^2-4 = k^2$, giải ra được $ t = 2$ hoặc $t = -2$.

$0=abc-a-b-c +2 = a(bc-1) – b-c+2 \geq bc-b-c+1 = (b-1)(c-1)$. Suy ra $b = c=1$ (vô lý).
$0=abc-a-b-c-2 \geq (b-1)(c-1) – 4\Rightarrow (b-1)(c-1) \leq 4$.
Nếu $(b-1)(c-1) = 4$ thì $b = 2, c=5$; $b = 3, c=3$; $b=5, c=2$. Trong các trường hợp này thì $a=1$.
Nếu $ a= 1, b = 2, c = 5$ giải được $(x, y, z) = (\dfrac{1}{3}, \dfrac{3}{2},2)$.
Nếu $a = 1, b = 3, c = 3$ thì $(x, y, z) = (\dfrac{1}{2},2,1)$.
Nếu $a = 1, b = 5, c = 2$ thì $(x, y, z) = (\dfrac{2}{3}, 3,2)$.
Nếu $(b-1)(c-1) = 3 \Rightarrow bc= b+c +2 = abc-a = a(bc-1) \Rightarrow bc-1|bc \Rightarrow bc = 1, a = 1$. (loại)
Khi $(b-1)(c-1) =2 \Rightarrow a = b = c = 2$, giải ra được $(x, y, z) = (1, 1, 1)$.

Trên đây là một số bài toán liên quan đến số hữu tỉ, vô tỉ, hi vọng các em có thêm kinh nghiệm để làm bài trong các tình huống này. Sau đây là một số bài tập rèn luyện.

Bài 1.  Tìm một đa thức hệ số nguyên nhận $\alpha = 2 + \sqrt[3]{2} + \sqrt[3]{4}$ làm nghiệm. Chứng minh $\alpha$ là số vô tỷ.
Bài 2.  Cho các số $a, b$ sao cho $a – \sqrt{ab}$ và $b-\sqrt{ab}$ đều là các số hữu tỉ. Chứng minh rằng $a, b$ cũng là các số hữu tỉ.
Bài 3. Ta nói các căp số $(\mathrm{a}, \mathrm{b}) a \neq b$, là có tính chất $\mathrm{P}$ nếu $a^{2}+b \in Q$ và $b^{2}+a \in \mathbb{Q}$.
Chứng minh rằng:
a) Các số $a=\dfrac{1+\sqrt{2}}{2}, b=\dfrac{1-\sqrt{2}}{2}$ là các số yô tỷ có tính chất $\mathrm{P}$.
b) Nếu $(\mathrm{a}, \mathrm{b})$ có tính chất $\mathrm{P}$ và $a+b \in \mathbb{Q} \backslash{1}$ thì $a, b$ à các số hũu tỷ.
c) Nếu $(\mathrm{a}, \mathrm{b})$ có tính chất $\mathrm{P}$ và $\dfrac{a}{b} \in \mathbb{Q}$ thì $\mathrm{a}, \mathrm{b}$ là các số hũu tỷ.
Bài 4.  Với mỗi số hữu tỷ $q$ đặt $V_q = {x \in \mathbb{Q}|x^3-2015x =q}$.

a)Tìm $q$ sao cho $V_q$ có là tập rỗng và $V_q$ có đúng một phần tử.
b) Gọi $S(V_q)$ là số phần tử của $V_q$, tìm tất cả các giá trị của $S(V_q)$.
Bài 5.
a) Cho số thực $x$ thỏa $x^2+x$ và $x^3+2x$ là số hữu tỷ. Chứng minh $x$ cũng là số hữu tỷ.
b) Chứng minh rằng tồn tại số vô tỷ $x$ sao cho $x^2+x$ và $x^3-2x$ là hữu tỷ.

Hết

Chuyên đề hình học: Bổ đề Eriq và ứng dụng

BỔ ĐỀ ERIQ VÀ ỨNG DỤNG (Trích tập san Star số 3)

Trương Tuấn Nghĩa – Lớp 12 Trường ĐHKHTN ĐHQG HN

Giới thiệu.

Bổ đề $ERIQ$ được đặt tên bởi tác giả Kostas Vittas trên diễn đàn AoPS với nick name vittasko. (là các chữ viết tắt của cụm từ $Equal$ $Ratios$ $In$ $Quadrilateral$). Nội dung bổ đề:

Cho tứ giác $ABCD$, lấy các điểm $M,N$ nằm trên cạnh $AD,BC$ sao cho
$\dfrac{MA}{MD}=\dfrac{NB}{NC}.$
Khi đó, trung điểm của $AB,MN,CD$ thẳng hàng.

Chứng minh.
Gọi $X,Y,Z$ là trung điểm của $AB,MN,CD$. Lấy $P,Q$ nằm trên $XM,XN$ sao cho $DP,CQ\parallel AB.$

Khi đó, theo định lý Thales, ta có $\frac{MA}{MD}=\frac{AX}{DP}=\frac{MX}{MP};\text{ }\frac{NB}{NC}=\frac{AY}{CQ}=\frac{NX}{NQ}.$ Suy ra
$DP=CQ;$ $\frac{MX}{MP}=\frac{NX}{NQ}$ hay $MN\parallel PQ$.
Do $DP=CQ;DP\parallel CQ$ nên $PCQD$ là hình bình hành hay $Z$ là trung điểm $PQ$. \

Kết hợp với $Y$ là trung điểm của $MN$, ta có $X,Y,Z$ thẳng hàng.

Nhận xét. Ta có thể chứng minh $X,Y,Z$ là các điểm chia cùng tỉ lệ trên $AB,MN,CD$ thẳng hàng bằng cách tương tự. Tiếp theo, ta sẽ đến với một số các mở rộng và ứng dụng của bổ đề trên.

Ứng dụng

Bài 1.  Cho tứ giác $ABCD$, lấy $M,N$ nằm trên cạnh $AD,BC$ sao cho $\frac{MA}{MD}=\frac{NB}{NC}.$ Lấy các điểm $X,Y,Z$ sao cho các tam giác $XAB,YMN,ZCD$ đồng dạng và $X,Y,Z$ lần lượt nằm trên các nửa mặt phẳng bờ $AB$ không chứa $C$, $MN$ không chứa $D$ và $CD$ chứa $A$. Chứng minh rằng $X,Y,Z$ thẳng hàng.
Lời giải.
Lấy $P,Q\in XM,XN$ sao cho $DP\parallel XA,CQ\parallel XB$.

Theo định lý Thales, $DP=XA.\frac{MD}{MA},CQ=XB.\frac{NB}{NC}$ mà $\frac{MA}{MD}=\frac{NB}{NC}$ nên $DP=CQ$
Mặt khác vì $\angle AXB=\angle CZD$ nên $\angle ZDP=\angle ZCQ.$
Do đó, $\vartriangle ZDP=\vartriangle ZCQ(c.g.c)$ dẫn tới $\angle PZD=\angle QZC$ hay $\angle CZD=\angle PZQ.$
Vì $DP\parallel XA,CQ\parallel XB$ nên $\frac{XM}{MP}=\frac{XN}{NQ}(=\frac{MA}{MD})$ nên $MN\parallel PQ$.
Lấy $Y’\in XZ$ sao cho $\frac{XY’}{Y’Z}=\frac{XM}{MP}=\frac{XN}{NQ}.$
Theo định lý Thales, $\frac{XY’}{Y’Z}=\frac{XM}{MP}=\frac{XN}{NQ}$ nên $$\begin{aligned}
& Y’M\parallel ZP,Y’N\parallel ZQ \
& Y’M=Y’N(=ZP.\frac{XY’}{XZ}=ZQ.\frac{XY’}{XZ}) \
\end{aligned}$$
Hay $\angle MY’N=\angle MYN,Y’M=Y’N.$
Do đó, $Y’\equiv Y$ hay $X,Y,Z$ thẳng hàng.

Bài 2. Cho tứ giác $ABCD$ có phân giác trong của các góc $\angle A,\angle B,\angle C,\angle D$ đồng quy tại $I$. $AD$ cắt $BC$ tại $E$, $AB$ cắt $CD$ tại $F$. Gọi $M,N$ là trung điểm $AC,EF.$ Chứng minh rằng $M,N,I$ thẳng hàng.
Lời giải.

Gọi $P,Q$ là giao điểm của đường thẳng qua $I,$ vuông góc với $IB$ với $BA,AC.$
Đầu tiên, dễ thấy $I$ là giao 3 phân giác $\vartriangle ABE$.
Do $BI$ là phân giác $\angle ABC$ nên $\vartriangle BPQ$ cân tại $B$ hay $I$ là trung điểm $PQ.$


Ta có $\angle BPQ=90{}^\circ -\frac{\angle ABE}{2}=\frac{\angle AEB}{2}+\frac{\angle BAE}{2},\angle IAB=\frac{\angle BAE}{2}$ nên $\angle PIA=\frac{\angle AEB}{2}.$
Tương tự thì $\angle EIQ=\frac{\angle BAE}{2}.$
Do đó, $\vartriangle PIA\sim \vartriangle QEA(g.g)$ nên $PA.QE=PI.QI.$
Hoàn toàn tương tự, $PF.QC=PI.QI.$
Vậy ta có $\frac{PA}{FA}=\frac{QC}{QE}$ nên theo bổ đề $ERIQ$, $M,I,N$ lần lượt là trung điểm của $PQ,AC,EF$ thẳng hàng.

Bài 3. Cho tứ giác $ABCD$ nội tiếp, không là hình thang. Gọi $E,F$ là giao điểm của các cặp đường thẳng $(AB,CD);(AD,BC).$ Giả sử phân giác của góc $\angle AEC,\angle AFB$ cắt nhau tại $I$. Gọi $M,N$ lần lượt là trung điểm của $AC,BD$. Chứng minh rằng $I\in MN.$

Lời giải.
Giả sử các điểm có vị trí như hình vẽ, các trường hợp khác tương tự.


Gọi $P,Q$ lần lượt là giao điểm của $FI$ với $AB,CD$.
Do $\angle ABC+\angle CDA=180^\circ $ nên $\angle FAB=\angle FCD$ nên $\triangle FAB \backsim \triangle FCD(g.g)$ () và $\angle EPQ=\angle FAB+\angle AFI=\angle FCD+\angle BFI=\angle EQP$
hay tam giác $EPQ$ cân tại $E$.
Mà $EI$ là phân giác $\angle AED$ nên $I$ là trung điểm $PQ$.
Mặt khác theo (
), $\frac{FA}{FB}=\frac{FC}{FD}$ nên theo tính chất đường phân giác, $\frac{AP}{PB}=\frac{CQ}{QD}.$
Do đó theo bổ đề $ERIQ$, trung điểm $AC,BD,PQ$ thằng hàng hay $I\in MN$. (đpcm)

Bài 4. (AOPS). Cho $\vartriangle ABC$, trực tâm $H$,$P$ bất kỳ trên $BC$, $X$ bất kỳ trên $HP$. Gọi $E,F\ne A$ là giao điểm của đường tròn đường kính $AX$ với $CA,AB$. Tiếp tuyến tại $E,F$ của $(AEF)$ cắt nhau tại $T$. Đường thẳng qua $P$ vuông góc $BC$ cắt $CA,AB$ tại $Z,Y$. Gọi $L$ là trung điểm $ZY$. Chứng minh rằng $LT$ chia đôi $BC.$

Lời giải.
Trước hết, ta phát biểu và chứng minh hai bổ đề sau:
Bổ đề 1. Cho $\vartriangle ABC$, đường cao $BE,CF$. Gọi $M$ là trung điểm của $BC.$ Khi đó, $ME,MF$ là tiếp tuyến của $(AEF)$.
Bổ đề trên có thể chứng minh dễ dàng qua các phép cộng góc.
Bổ đề 2.Cho tứ giác $ABCD$, $AB$ cắt $CD$ tại $E$. Gọi $H,K$ là trực tâm của $\vartriangle EAD,\vartriangle EBC$. Khi đó, $HK$ là trục đẳng phương của 2 đường tròn đường kính $BD,AC$.
Chứng minh bổ đề
Gọi $M,N$ là hình chiếu của $B,C$ lên $EC,EB$. Khi đó, $MNBC$ là tứ giác nội tiếp nên $KN.KC=KM.KB.$

Mặt khác, $M,N$ lần lượt nằm trên đường tròn đường kính $BD,AC$ mà $KN.KC=KM.KB$ nên $K$ nằm trên trục đẳng phương của 2 đường tròn trên. Chứng minh tương tự, $HK$ là trục đẳng phương của đường tròn đường kính $BD$ và đường tròn đường kính $AC$.

Trở lại bài toán,


Gọi $M,N$ là giao điểm của $XF,XE$ với $CA,AB.$ Khi đó, theo bổ đề 1 dễ có $T$ là trung điểm của $MN$ nên theo bổ đề $ERIQ$, ta chỉ cần chứng minh $\frac{BN}{BZ}=\frac{CM}{CY}.$
Gọi $U,V$ là hình chiếu của $N,M$ lên $BC.$ Theo bổ đề 2 thì $HX$ là trục đẳng phương của đường tròn đường kính $MB,NC.$ Dễ thấy $U,V$ lần lượt nằm trên đường tròn đường kính $CN,BM$ nên và $P$ nằm trên $HX,BC$ nên ta có $PU.PC=PV.PB$ hay $\frac{PB}{PU}=\frac{PC}{PV}$, và theo định lý Thales thì
$\frac{BN}{BZ}=\frac{CM}{CY}$ .
Vậy ta thu được $LT$ chia đôi $BC.$

Bài 5. Cho $\vartriangle ABC$, $P$ bất kỳ trên $BC$, $J$ là trung điểm của $AP$. Gọi $E,F$ là giao điểm của $(J,JA)$ với $CA,AB.$ Gọi $L$ là tâm đường tròn ngoại tiếp $\vartriangle JEF$. Chứng minh rằng khi $P$ di chuyển trên $BC$ thì $L$ chuyển động trên đường thẳng cố định.

Lời giải
Trước hết ta chứng minh bổ đề sau:
Cho $\vartriangle ABC$, lấy điểm $M$ cố định trên $BC,P$ bất kỳ trên $BC.$ Gọi $E,F$ là hình chiếu của $P$ lên $CA,AB$, $K,L$ là hình chiếu của $M$ lên $CA,AB$. Khi đó, tỉ số $\frac{EK}{FL}$ không phụ thuộc vào vị trí của $P$ trên $BC.$

Chứng minh.
Gọi $X,Y$ là hình chiếu của $M,P$ lên $PF,MK$. Khi đó,
$$\begin{aligned}
& MX=LF=MP.\cos \angle XMP=MP.cos\angle ABC; \
& YP=KE=MP.\cos \angle YPM=MP.\cos \angle ACB. \
\end{aligned}$$
Do đó, $\frac{EK}{FL}=\frac{\cos \angle ACB}{\cos \angle ABC}.$

Trở lại bài toán,


Lấy $M,N$ cố định trên $BC.$ $X,Z$ là hình chiếu của $M$ lên $AB,AC;$ $Y,T$ là hình chiếu của $N$ lên $AB,AC.$ Khi đó, theo bổ đề 1 thì dễ có được $\frac{YF}{YX}=\frac{TE}{TZ}.$ (1)
Do $J$ là tâm đường tròn ngoại tiếp $\vartriangle AEF$ nên $\angle FJE=2.\angle BAC.$ Mà $L$ là tâm đường tròn ngoại tiếp của $\vartriangle JEF$ nên $\angle FLE=360{}^\circ -4.\angle BAC.$
Theo (1) và bổ đề $ERIQ$ thì các đỉnh của tam giác cân có đáy $FE,YT,XZ$ và có góc ở đỉnh là $360{}^\circ -4.\angle BAC$ thì thẳng hàng mà $M,N$ cố định nên $L$ nằm trên đường thẳng cố định. (đpcm)

Bài 6.  (Nguyễn Văn Linh) Cho $\vartriangle ABC$, đường cao $AD$, $K\in AD.$ Gọi $E,F$ lần lượt là giao điểm của $BK,CK$ với $CA,AB.$ Giả sử $DE,DF$ cắt lại đường tròn ngoại tiếp $\vartriangle ABD;\vartriangle ACD$ tại $M,N$. Gọi $T$ là trung điểm của $MN.$ Chứng minh rằng $AT$ chia đôi đoạn thẳng $EF.$

Lời giải
Gọi $BP,CQ$ là đường cao của $\vartriangle ABC$, đường thẳng qua $A$ song song $BC$ cắt $DE,DF$ tại $K,L.$ Theo kết quả quen thuộc $DF,DE$ đối xứng nhau qua $AD$ và $DQ,DP$ đối xứng nhau qua $AD.$ Nên $A$ là trung điểm của $KL.$


Khi đó, theo bổ đề $ERIQ,$ ta chỉ cần chứng minh $\frac{NL}{NF}=\frac{MK}{ME}.$
Ta có, $A,M,P,D,Q$ nằm trên đường tròn và $A,N,Q,D,C$ nằm trên đường tròn. (1) \
Do đó, $\angle NAQ=\angle NDQ,\angle MAP=\angle MDP.$ Do $DF,DE$ đối xứng nhau qua $AD$ và $DQ,DP$ đối xứng nhau qua $AD,$nên $\angle QDF=\angle PDE.$
Từ (1), ta cũng có
$\angle AQN=\angle ADN=\angle ADM=\angle APM.$
Do đó, $\vartriangle ANQ\sim \vartriangle AMP.$ (2) \
Mặt khác, $\frac{FL}{AL}=\frac{\sin LFA}{\sin LAF};\frac{KA}{KE}=\frac{\sin KAE}{\sin KEA}.$ Vì $AK=AL;\angle FAL=\angle ABC;\angle EAK=\angle ACB,$ nên
$$\begin{aligned}
\frac{FL}{AL}.\frac{KA}{KE} &=\frac{\sin LFA}{\sin FAL}.\frac{\sin KAE}{\sin KEA}=\frac{FL}{KE} \
& =\frac{\sin LFA}{\sin KEA}.\frac{\sin KAE}{\sin FAL}=\frac{\sin ACB}{\sin ABC}.\frac{\sin LFA}{\sin KEA}=\frac{AB}{AC}.\frac{\sin LFA}{\sin KEA}. \
\end{aligned}$$
Ta lại có
$$\frac{\sin LFA}{\sin KEA}=\frac{\sin NFA}{\sin NAF}.\frac{\sin MAP}{\sin MEA}=\frac{AN}{FN}.\frac{ME}{MA}=\frac{AN}{AM}.\frac{ME}{FN}=\frac{AQ}{AP}.\frac{ME}{FN}=\frac{AC}{AB}.\frac{ME}{FN}.$$ (do (2)). Vậy nên $$\frac{FL}{KE}=\frac{AB}{AC}.\frac{\sin LFA}{\sin KEA}=\frac{AB.AC}{AC.AB}.\frac{ME}{NF}=\frac{ME}{NF}.$$

Bài 7. (Chọn đội tuyển PTNK TPHCM) Cho $\vartriangle ABC$, trực tâm $H.$ Lấy điểm $M$ bất kỳ trên cung $BHC$ của $(BHC)$. Trên $BM,CM$ lấy các điểm $E,F$ sao cho $\angle ECA=\angle FBA=90{}^\circ .$ Chứng minh rằng khi $M$ chuyển động thì trung điểm $EF$ luôn nằm trên đường thẳng cố định.

Lời giải. Ở bài toán này, ta có hai hướng tiếp cận như sau:
Cách 1.
Gọi $N$ là giao điểm của $CE,BF.$ Lấy $P$ đối xứng với $N$ qua $BC$, $BP,CP$ lần lượt cắt $CE,BF$ tại $X,Y.$ Dễ dàng chứng minh $B,H,M,P,C$ nằm trên đường tròn.


Ta sẽ chứng minh $\frac{XE}{YF}$ không đổi khi $M$ chuyển động trên cung $BHC.$
Do $\angle BMC=\angle BNC=180{}^\circ -\angle BAC$ nên $\angle CME=\angle CNF$ hay 4 điểm $M,N,E,F$ nằm trên đường tròn nên $\angle CFY=\angle BEX.$ (1)
Mặt khác, do $B,H,M,P,C$nằm trên đường tròn nên $\angle YCF=\angle MCP=\angle XBE.$ (2)
Từ (1) và (2) suy ra $\vartriangle CYF\sim \vartriangle BXE(g.g)$. Do đó, $\frac{XE}{YF}=\frac{BX}{CY}$ không đổi.
Vậy $\frac{XE}{YF}$ không đổi khi $M$ chuyển động trên cung $BHC$ nên theo bổ đề $ERIQ$, trung điểm của $EF$ luôn nằm trên đường thẳng cố định. \medskip

Cách 2. Trước hết ta phát biểu và chứng minh bổ đề sau: \textbf{(IMO2009 Shortlist G4)} Cho tứ giác $ABCD$ nội tiếp đường tròn $(O).$ $AC$ cắt $BD$ ở $E,$ $AD$ cắt $BC$ tại $F.$ Gọi $M,N$ lần lượt là trung điểm của $AB,CD$. Khi đó, $EF$ tiếp xúc với đường tròn ngoại tiếp của $\vartriangle EMN.$
Chứng minh.
Gọi $I$ là trung điểm của $EF.$ Xét tứ giác toàn phần $AEBF.CD$ có $I,M,N$ lần lượt là trung điểm của các đường chéo $EF,AB,CD$ nên $I,M,N$ thẳng hàng.


Ta sẽ chứng minh $\overline{IM}.\overline{IN}=I{{E}^{2}}.$
Gọi $L,P,T$ lần lượt là giao điểm của $AB$ với $CD$, $EF$ với $AB,CD$. Khi đó,
$(LP,AB)=(LT,CD)=-1$
nên áp dụng hệ thức $Maclaurin$ và $ABCD$ là tứ giác nội tiếp, ta thu được
$\overline{LM}.\overline{LP}=\overline{LA}.\overline{LB}=\overline{LC}.\overline{LD}=\overline{LT}.\overline{LN}$
nên 4 điểm $M,P,N,T$ nằm trên đường tròn.
Do đó, $\overline{IM}.\overline{IN}=\overline{IP}.\overline{IT}.$
Mặt khác, ta lại có $(EF,PT)=-1$ nên theo $I{{E}^{2}}=\overline{IT}.\overline{IP}$.
Vậy $\overline{IM}.\overline{IN}=I{{E}^{2}}.$ Do đó, $EF$ là tiếp tuyến của đường tròn ngoại tiếp $\vartriangle EMN.$ (đpcm)

Trở lại bài toán,
Gọi $N$ là giao điểm của $CE,BF.$ Lấy $I,P,Q$ lần lượt là trung điểm của $BC,EF,MN.$

Theo lời giải thứ nhất, ta có 4 điểm $M,N,E,F$ nằm trên đường tròn nên theo bổ đề 4 thì $BC$ là tiếp tuyến của $(QCP)$ hay $I{{C}^{2}}=\overline{IQ}.\overline{IP}.$

Do đó, $I_I^{IC^2}:P\leftrightarrow Q.$ (1)
Mặt khác $V_{N}^{2}:Q\mapsto M$ mà $M$ chuyển động trên cung $BHC$ nên $Q$ chuyển động trên đường tròn $(\omega )$ cố định. (2)

Từ (1) và (2), ta thu được $P$ chuyển động trên đường thẳng ảnh của $(\omega )$ qua ${I}_{I}^{IC^2}:P\leftrightarrow Q.$

Nhận xét. Qua các bài toán trên, ta có thể thấy được ứng dụng của bổ đề $ERIQ$ trong các bài toán hình học. Sau đây sẽ là một số các bài toán luyện tập.

Bài tập tự giải.

  1. Cho $\vartriangle ABC$ nội tiếp $(O)$. Tiếp tuyến của $(O)$ tại $A$ cắt tiếp tuyến của $(O)$ tại $B,C$ lần lượt tại $E,F$. Gọi $M,N$ là trung điểm của $BF,CE$. Đường thẳng qua $O$ và vuông góc với $OA$ cắt $BC$ tại $S$. Chứng minh rằng $MN$ chia đôi $SO$.

  2. Cho $\vartriangle ABC,$ trực tâm $H$, trung tuyến $AM.$ $P$ bất kỳ trên $HM$. Đường tròn đường kính $AP$ cắt $CA,AB$ tại $E,F$. Tiếp tuyến tại $E,F$ của $(AEF)$ cắt nhau tại $T$. Chứng minh rằng $TB=TC.$

  3. Cho $\vartriangle ABC$, đường tròn $(K)$ đi qua $B,C$ cắt $CA,AB$ tại $E,F$. Gọi $H$ là giao điểm của $BE,CF.$ Lấy $P$ bất kỳ trên $BC$. Đường thẳng qua $P$ và song song với $AH$ cắt $CA,AB$ tại $X,Y.$Lấy $Q$ bất kỳ trên $HP.$ Đường thẳng qua $Q$ song song với $BE,CF$ cắt $CA,AB$ tại $X,Y,Z,T.$ \
    a) Chứng minh rằng 4 điểm $X,Y,Z,T$ nằm trên đường tròn $(L)$. \
    b) $KL$ cắt trung trực $PQ$ tại $Z$. Chứng minh rằng $\vartriangle ZPQ\sim \vartriangle KBC.$

  4. Cho $\vartriangle ABC$, $P$ bất kỳ trên $BC.$ Đường thẳng qua $P$ song song với $CA,AB$ cắt trung trực $BA,AC$ tại $M,N$. Chứng minh rằng khi $P$ chuyển động trên $BC$, tâm đường tròn ngoại tiếp của $\vartriangle MNP$ luôn nằm trên một đường thẳng cố định.

  5. (Việt Nam TST 2008) Cho $\triangle ABC$ nhọn không cân nội tiếp $(O).$ Với $k\in {{\mathbb{R}}^{+}},$ trên các đoạn phân giác $AD,BE,CF,$ lấy $M,N,P$ sao cho $\frac{AM}{AD}=\frac{BN}{BE}=\frac{CP}{CF}=k.$

Vẽ đường tròn $({{O}_{1}})$ đi qua $A,M$ và tiếp xúc với $OA;$

Vẽ đường tròn $({{O}_{2}})$ đi qua $B,N$ và tiếp xúc với $OB;$

vẽ đường tròn $({{O}_{3}})$ đi qua $C,P$ và tiếp xúc với $OC.$

Tìm tất cả các giá trị $k$ sao cho $(O_1),(O_2),(O_3)$ có đúng hai điểm chung.

  1. Cho tam giác $ABC$ nhọn không cân có điểm $D$ thay đổi trong tam giác sao cho $\angle ABD=\angle ACD,$ lấy $E\in AB,F\in AC$ sao cho $D$ là trực tâm tam giác $AEF.$ Chứng minh rằng:
    a) Trung tuyến đỉnh $D$ của tam giác $DEF$ luôn đi qua điểm cố định.
    b) Trung trực $EF$ luôn đi qua điểm cố định.
    c) Tâm đường tròn ngoại tiếp tam giác $(DEF)$ luôn thuộc đường cố định.
    d) Trục đẳng phương của $(BDE),(CDF)$ luôn đi qua một điểm cố định.

Tài liệu tham khảo.

  1. Nguyễn Văn Linh, Về bài 3 đề VMO 2016.
  2. Nguyễn Văn Linh, 2015, Định lý ERIQ, \url{https://nguyenvanlinh.wordpress.com
  3. Diễn đàn \url{artofproblemsolving.com/community
  4. Trần Quang Hùng, Các bài giảng đội tuyển.

Chuyên đề: Biến đổi biểu thức

RÚT GỌN BIẾN ĐỔI BIỂU THỨC CHỨA CĂN THỨC

Chuyên đề này đề cập tới các bài toán rút gọn biểu thức, chứng minh các đẳng thức, tính toán biểu thức,…Đây là chuyên đề quan trọng, rèn luyện kĩ năng biến đổi đại số cho các em, là kĩ năng ta sẽ dùng sau này.

Kiến thức là toàn bộ chương căn bậc hai, các hằng đẳng thức và kĩ năng biến đổi đã học ở lớp 8.

Các bạn có thể xem trước các bài cơ bản ở đây.

Dạng 1. Tính toán rút gọn

Ví dụ 1. Đặt $x = \sqrt{2}+\sqrt{3}$.
a) Chứng minh rằng $x^4 – 10x^2 + 1 = 0$.
b) Tìm giá trị của biểu thức $P(x) = (x^6 – 11x^4 + 11x^2 + 1)^{2019}$.

Lời giải

 

 

 

 

 

 

 

Ví dụ 2.  Cho $x$ thỏa $x \geq 2$. Rút gọn biểu thức $$A = \dfrac{{{x^3} – 3x + \left( {{x^2} – 1} \right)\sqrt {{x^2} – 4} – 2}}{{{x^3} – 3x + \left( {{x^2} – 1} \right)\sqrt {{x^2} – 4} + 2}}$$

Lời giải

Ví dụ 3.

a) Chứng minh rằng với mọi số nguyên dương n ta có: $$1 + \dfrac{1}{{{n^2}}} + \dfrac{1}{{{{\left( {n + 1} \right)}^2}}} = {\left( {1 + \dfrac{1}{n} – \dfrac{1}{{n + 1}}} \right)^2}$$
b) Tính tổng $$S = \sqrt {1 + \dfrac{1}{{{1^2}}} + \dfrac{1}{{{2^2}}}} + \sqrt {1 + \dfrac{1}{{{2^2}}} + \dfrac{1}{{{3^2}}}} + \cdots + \sqrt {1 + \dfrac{1}{{{{2021}^2}}} + \dfrac{1}{{{{2022}^2}}}} $$

Lời giải

Ví dụ 4. Rút gọn biểu thức: $$A = \dfrac{1}{{2\sqrt 1 + 1\sqrt 2 }} + \dfrac{1}{{3\sqrt 2 + 2\sqrt 3 }} + \cdots + \dfrac{1}{{2019\sqrt {2018} + 2018\sqrt {2019} }}$$

Lời giải

Dạng 1. Chứng minh đẳng thức

Ví dụ 5. Cho $a, b \ge 0, a^2>b$. Chứng minh $$\sqrt{a+\sqrt{b}}=\sqrt{\dfrac{a+\sqrt{a^2-b}}{2}}+\sqrt{\dfrac{a-\sqrt{a^2-b}}{2}}$$ và $$\sqrt{a-\sqrt{b}}=\sqrt{\dfrac{a+\sqrt{a^2-b}}{2}}-\sqrt{\dfrac{a-\sqrt{a^2-b}}{2}}$$

Lời giải

Ví dụ 6. Cho $a, b >0, c \neq 0$. Chứng minh rằng:
$$ \dfrac{1}{a} + \dfrac{1}{b} + \dfrac{1}{c} = 0 \Leftrightarrow \sqrt {a + b} = \sqrt {a + c} + \sqrt {b + c} $$

Lời giải

Ví dụ 7. Cho $xy + \sqrt {\left( {1 + {x^2}} \right)\left( {1 + {y^2}} \right)} = a > 1$. Tính $S = x\sqrt {1 + {y^2}} + y\sqrt {1 + {x^2}} $.

Lời giải

Ví dụ 8. Đặt $a_n = \sqrt[4]{2} + \sqrt[n]{4}, n = 2, 3…$. Chứng minh rằng $$ \dfrac{1}{a_5}+\dfrac{1}{a_6}+\dfrac{1}{a_{12}}+\dfrac{1}{a_{20}} = \sqrt[4]{8} $$

Lời giải

Ví dụ 9.  Chứng minh rằng nếu $\sqrt[3]{a} + \sqrt[3]{b} + \sqrt[3]{c} = \sqrt[3]{{a + b + c}}$ thì với mọi số nguyên dương lẻ n ta có $\sqrt[n]{a} + \sqrt[n]{b} + \sqrt[n]{c} = \sqrt[n]{{a + b + c}}$.

Lời giải

Dạng 3. Hữu tỉ và vô tỉ

Ví dụ 10. 

a) Chứng minh rằng $\sqrt{2}$ là số vô tỉ.

b) Cho $n$ và số tự nhiên và $m$ là số tự nhiên thỏa $n^2 < m < (n+1)^2$. Chứng minh $\sqrt{m}$ là một số vô tỉ.

Lời giải

Ví dụ 11. Chứng minh số
$A=\sqrt[3]{7+5\sqrt{2}}+\sqrt[3]{7-5\sqrt{2}}$ là một số nguyên.

Lời giải

Ví dụ 12. 

a) Chứng minh rằng nếu $a, b$ là các số hữu tỉ thỏa $a+b\sqrt{2} = 0$ thì $a = b= 0$.

b) Tìm các số $a, b$ hữu tỉ thỏa $\sqrt{a} +\sqrt{b} = \sqrt{2+\sqrt{3}}$.

 

Bài tập rèn luyện.

Bài 1. Với mọi $x \ge 2$. Chứng minh rằng $$\sqrt{\sqrt{x}+\sqrt{\dfrac{x^2-4}{x}}}+\sqrt{\sqrt{x}-\sqrt{\dfrac{x^2-4}{x}}}=\sqrt{\dfrac{2x+4}{\sqrt{x}}}$$

Bài 2. Rút gọn $A=\sqrt{\dfrac{1}{x^2+y^2}+\dfrac{1}{(x+y)^2}+\sqrt{\dfrac{1}{x^4}+\dfrac{1}{y^4}+\dfrac{1}{(x^2+y^2)^2}}}$

Bài 3. Cho $x,y<0$. Chứng minh $|\sqrt{xy}-\dfrac{x+y}{2}|+|\dfrac{x+y}{2}+\sqrt{xy}|=|x|+|y|.$
Bài 4. Cho các số $x,y,z>0$ và đôi một phân biệt. Chứng minh giá trị của $P$ không phụ thuôc vào $x,y,z$ với
$$P=\dfrac{x}{(\sqrt{x}-\sqrt{y})(\sqrt{x}-\sqrt{z})}+\dfrac{y}{(\sqrt{y}-\sqrt{z})(\sqrt{y}-\sqrt{x})}+\dfrac{z}{(\sqrt{z}-\sqrt{x})(\sqrt{z}-\sqrt{y})}.$$
Bài 5.  Cho $a=\sqrt{2}+\sqrt{7-\sqrt[3]{61+46\sqrt{5}}}+1$.

a) Chứng minh: $a^4-14a^2+9=0$.
b) Cho $f(x)=x^5+2x^4-14x^3-28x^2+9x+19$. Tính $f(a).$

Bài 6.  Cho $a=\sqrt[3]{38+17\sqrt{5}}+\sqrt[38]{38-17\sqrt{5}}$ và $f(x)=(x^3+3x+2018)^{2018}$. Tính $f(a).$
Bài 7.  Cho $x=1+\sqrt[3]{2}+\sqrt[3]{4}$. Tính $x^5-4x^4+x^3-x^2-2x+2018.$

Bài 8. Cho $f(n)=\dfrac{4n+\sqrt{4n^2-1}}{\sqrt{2n+1}+\sqrt{2n-1}}, n \in \mathbb{N}^*$. Tính $f(1)+f(2)+…+f(2018)$. %NTK

Bài 9.  Cho $f(n)=\dfrac{2n+1+\sqrt{n(n+1)}}{\sqrt{n}+\sqrt{n+1}}$. Tính $f(1)+f(2)+…+f(n).$ %NTK
Bài 10. Cho $x,y,z >0$ thoả $xyz=4$. Tính giá trị biểu thức $$A=\dfrac{\sqrt{x}}{\sqrt{xy}+\sqrt{x}+2}+\dfrac{\sqrt{y}}{\sqrt{yz}+\sqrt{y}+1}+\dfrac{2\sqrt{z}}{\sqrt{xz}+2\sqrt{z}+2}.$$

Bài 11.  Cho các số dương $x,y,z$ thoả $\begin{cases} x+y+z=2&\\\sqrt{x}+\sqrt{y}+\sqrt{z}=2 \end{cases}$. Tính $$A=\sqrt{(1+x)(1+y)(1+z)}\left(\dfrac{\sqrt{x}}{x+1}+\dfrac{\sqrt{y}}{y+1}+\dfrac{\sqrt{z}}{z+1}\right).$$

Bài 12.  Cho các số $abc \ne 0$ thoả $a+b+c=0$. Chứng minh $$\sqrt{\dfrac{1}{a^2}+\dfrac{1}{b^2}+\dfrac{1}{c^2}}=\big|\dfrac{1}{a}+\dfrac{1}{b}+\dfrac{1}{c}\big|$$

Bài 13.  Cho $a,b,c>0$ thoả $a\sqrt{1-b^2}+b\sqrt{1-c^2}+c \sqrt{1-a^2}=\dfrac{3}{2}.$\ Chứng minh $a^2+b^2+c^2=\dfrac{3}{2}.$
Bài 14.  Tìm tất cả các số thực $a,b,c$ thoả $\sqrt[3]{a-b}+\sqrt[3]{b-c}+\sqrt[3]{c-a}=0.$ %105-38
Bài 15. Cho các số $a_1, a_2,…,a_n$ thoả $a_1=1, a_{n+1}=\dfrac{\sqrt{3}+a_n}{1-\sqrt{3}a_n}$. Tính $a_{2020}$.
Bài 16.  Chứng minh rằng nếu $\sqrt{x^2+\sqrt[3]{x^4y^2}}+\sqrt{y^2+\sqrt[3]{x^2y^4}}=a$ thì $$\sqrt[3]{x^2}+\sqrt[3]{y^2}=\sqrt[3]{a^2} $$

Phương trình nghiệm nguyên – Phương pháp sử dụng tính chất chia hết

1. Sử dụng tính chẵn, lẻ

Ví dụ 1: Tìm nghiệm nguyên của phương trình: $x^2 -2y^2 =5$ $(1)$.

Giải

Vì $x$, $y$ nguyên nên từ phương trình $(1)$ suy ra $x$ là số lẻ.

Thay $x=2k+1\ (k\in \mathbb{Z})$ vào $(1)$ ta được

$4k^2 +4k +1 -2y^2 =5 \Leftrightarrow 2(k^2 + k -1) = y^2 \Rightarrow y$ là số chẵn.

Đặt $y=2t\ (t\in \mathbb{Z})$ ta có $2(k^2 +k -1) = 4t^2 \Leftrightarrow k(k+1) = 2t^2 +1$ $(*)$

Vì $k(k+1)$ là số chẵn mà $2t^2 +1$ là số lẻ nên phương trình $(*)$ vô nghiệm.

Vậy phương trình $(1)$ vô nghiệm

Ví dụ 2: Tìm nghiệm nguyên của phương trình $(2x+5y+1)(2^{|x|} + x^2 +x +y) =105$ $(2)$.

(Trích đề thi HSG lớp 9 TP. Hà Tĩnh năm 2006 – 2007)

Giải

Ở bài này ta thấy vế trái là tích của hai số nguyên mà vế phải là số lẻ nên nó phải là tích của hai số nguyên lẻ nên ta có thể sử dụng tính chất chẵn lẻ như sau:

Vì $105$ là số lẻ nên $2x + 5y +1$, $2^{|x|} + x^2 +x +y$ là các số lẻ. Suy ra $y$ là số chẵn , mà $x^2 +x = x(x+1)$ là số chẵn nên $2^{|x|}$ là số lẻ suy ra $x=0$

Thay $x=0$ vào $(2)$ ta được $(5y+1)(y+1) = 105 \Leftrightarrow 5y^2 + 6y -104 =0\Leftrightarrow y=4$ (vì $y$ là số chẵn). Do đó $y=4$. Vậy phương trình có nghiệm nguyên là $(0;4)$.

Ví dụ 3: Giải phương trình nghiệm nguyên $|19x + 15y| + 1975 = |19y + 5x| + 2016^x$ $(3)$.

Giải

Biến đổi phương trình $(3)$ ta được:

$1975 – 2016^x = (|19y + 5x| + 19y + 5x) – (|19x + 5y| + 19x + 5y) + 14(x-y)$.

Vì $|a| +a$ là số chẵn với mọi giá trị nguyên của $a$ nên vế phải là số chẵn do đó $1975 – 2016^x$ phải là số chẵn suy ra $2016^x$ là số lẻ suy ra $x=0$.

Thay $x=0$ vào phương trình $(3)$ ta được $|5y| + 1975 = |19y| +1 \Leftrightarrow 14|y| = 1974 \Leftrightarrow y=141$ hoặc $y=-141$.

Vậy phương trình có hai nghiệm nguyên $(x;y)$ là $(0;141)$ và $(0;-141)$.

Ví dụ 4: Tìm nghiệm nguyên của phương trình $x + x^2 + x^3 = 4y^2 +4$ $(4)$.

Giải

Ta có: $(4) \Leftrightarrow 1+x+x^2 +x^3= 4y^2 +4y +1\Leftrightarrow (x+1)(x^2+1) = (2y+1)^2$ $(*)$

Dễ thấy $(2y +1)^2$ lẻ suy ra $x+1$ và $x^2 +1$ là hai số lẻ. Giả sử $(x+1, x^2 +1) = d$ suy ra $d$ lẻ.

Mặt khác $x+1 \ \vdots \ d \Rightarrow 1-x^2 \  \vdots \ d$, kết hợp với $x^2 +1 \  \vdots \ d$ ta có $1-x^2 + 1+x^2\   \vdots \ d \Rightarrow 2\  \vdots \  d\Rightarrow d=1$ (vì $d$ lẻ)

Vì $(x+1)(x^2 +1)$ là số chính phương (theo $(*)$) và $(x+1,x^2+1)=1$ nên $x+1$ và $x^2 +1$ đều là số chính phương.

Dễ thấy $x^2$ và $x^2 +1$ là 2 số tự nhiên liên tiếp mà đều là số chính phương nên $x=0$.

Khi đó theo $(4)$ thì $y=0$ hoặc $y=-1$.

Vậy nghiệm của phương trình là $(0;0)$ hoặc $(0;-1)$

Ví dụ 5: Chứng tỏ phương trình: $x^4 + y^4 + z^4 + t^4 + k^4 =2015$ không có nghiệm nguyên.

Giải

Nếu $x$ là số chẵn thì $x^4 \  \vdots \ 16$.

Nếu $x$ là số lẻ thì $x^2 : 8$ dư $1$ nên $x^4 = (8k+1)^2 : 16$ dư $1$.

Như vậy mỗi số $x^4$, $y^4$, $z^4$, $t^4$, $k^4$ chia cho $16$ dư $1$ hoặc $0$ nên $x^4+y^4+z^4+t^4+k^4$ chia cho $16$ có số dư không lớn hơn $5$ còn vế phải $2015$ chia cho $16$ dư $15$.

Vậy phương trình không có nghiệm nguyên.

2. Sử dụng tính chất chia hết

Ví dụ 6: Chứng minh rằng không tồn tại các số nguyên $x$; $y$; $z$ thỏa mãn

$$x^{3}+y^{3}+z^{3}=x+y+z+2017\ (6)$$

Giải

$(6) \Leftrightarrow\left(x^{3}-x\right)+\left(y^{3}-y\right)+\left(z^{3}-z\right)=2017$ . Vì $x^{3}-x=(x-1) x(x+1)$  là tích của $3$ số nguyên liên tiếp nên chia hết cho $6$, tương tự $y^3 -y$, $z^3 -z$ cũng chia hết cho $6$ nên vế trái chia hết cho $6$ mà $2017$ không chia hết cho $6$ nên phương trình $(6)$ vô nghiệm.

Vậy không tồn tại các số nguyên $x$; $y$; $z$ thỏa mãn $x^{3}+y^{3}+z^{3}=x+y+z+2017$

Ví dụ 7: Tìm nghiệm nguyên của phương trình: $x^2y -5x^2 -xy -x +y -1=0$ $(7)$

(Trích đề thi HSG lớp $9$ huyện Can Lộc, Hà Tĩnh)

Giải

Đây là phương trình $2$ ẩn mà bậc đối với $y$ là bậc nhất nên ta dễ dàng biểu thị $y$ theo $x$ và ta có cách giải như sau:

$(7) \Leftrightarrow y=\dfrac{5 x^{2}+x+1}{x^{2}-x+1}\left( \text{do } x^{2}-x+1>0\right) \Rightarrow y=5+\dfrac{6 x-4}{x^{2}-x+1}$

Ta có $y \in \mathbb{Z}   \Leftrightarrow(6 x-4)\ \vdots \ \left(x^{2}-x+1\right) $

$\Leftrightarrow 2(3 x-2) \vdots\left(x^{2}-x+1\right) $

$\Leftrightarrow\left[\begin{array}{l} 2 \ \vdots \ \left(x^{2}-x+1\right) \\ 3 x-2 \ \vdots \ \left(x^{2}-x+1\right) \end{array}\right.$

(vì $x^{2}-x+1=x(x-1)+1$ là số lẻ).

  • TH1: $2:\left(x^{2}-x+1\right)$
    $\Leftrightarrow x^{2}-x+1=\pm 1$ (vì $.x^{2}-x+1$ lẻ)

$\Leftrightarrow x=0 ; x=1$ (thỏa mãn $x$ nguyên).

$+$ Với $x=0 \Rightarrow y=1$
$+$ Với $x=1 \Rightarrow y=7$

  • TH2: $(3 x-2):\left(x^{2}-x+1\right)\ (*)$
    $\Rightarrow x(3 x-2)\ \vdots \ \left(x^{2}-x+1\right)$
    $\Rightarrow\left(3 x^{2}-2 x\right)\ \vdots \ \left(x^{2}-x+1\right)$
    $\Rightarrow\left(3 x^{2}-3 x+3+x-3\right)\ \vdots \ \left(x^{2}-x+1\right)$
    $\Rightarrow(x-3) \vdots\left(x^{2}-x+1\right)$
    $\Rightarrow(3 x-9) \vdots\left(x^{2}-x+1\right) \ (**)$

Từ $(*)$ và $(**)$ ta được $7 \vdots\left(x^{2}-x+1\right)\Rightarrow x^2 -x+1$ bằng một trong các giá trị $-7$; $7$; $1$; $-1$.

Từ đây ta được các nghiệm: $(3;7)$, $(0;1)$, $(1;7)$.

Thử lại ta thấy phương trình $(7)$ có các nghiệm nguyên $(x;y)$ là $(3;7)$, $(0;1)$, $(1;7)$.

Ví dụ 8: Tồn tại hay không một số nguyên $n$ thỏa mãn $n^3 + 2015n=2014^{2016} +1$?

Giải

Giả sử tồn tại số nguyên $n$ thỏa mãn phương trình $n^3 + 2015n=2014^{2016} +1$, suy ra:

$$n^3 -n +2016n = 2014^{2016}+1$$

$\Leftrightarrow (n-1)n(n+1)+2016n=2014^{2016}+1$

Do $(n-1)n(n+1)$ là tích của ba số nguyên liên tiếp  nên chia hết cho $3$ và $2016\, \vdots \, 3$ nên $n^3-n+2016n \, \vdots \, 3$ hay $n^3 + 2015\, \vdots \, 3$.

Mặt khác $2014$ chia $3$ dư $1$ nên $2014^{2016}$ chia $3$ dư $1\Rightarrow 2014^{2016}$ chia $3$ dư $1\Rightarrow 2014^{2016}+1$ chia $3$ dư $2$

Từ đó ta thấy điều mâu thuẫn. Vậy không tồn tại số nguyên $n$ thỏa mãn phương trình.

Ví dụ 9: Tồn tại hay không hai số nguyên dương $a$ và $b$ thỏa mãn $a^3 + b^3 =2013$?

Giải

Giả sử tồn tại hai số nguyên dương $a$ và $b$ thỏa mãn $a^3 + b^3 =2013$.

Ta có: $(a+b)^{3}=a^{3}+b^{3}+3 a b(a+b)$
Vì $a^{3}+b^{3}=2013\, \vdots\, 3 \Rightarrow a^{3}+b^{3}+3 a b(a+b)\, \vdots\, 3$
$\Leftrightarrow(a+b)^{3}\, \vdots\, 3 \Rightarrow a+b \, \vdots\, 3 \Rightarrow(a+b)^{3}\, \vdots\, 9$
$\Rightarrow 2013=a^{3}+b^{3}=(a+b)^{3}-3 a b(a+b)\, \vdots \, 9$ (vô lý).
Vậy không tồn tại hai số nguyên dương $a$ và $b$ thỏa mãn $a^3 + b^3 =2013$,

Ví dụ 10: Giải phương trình nghiệm nguyên $x^2(x-y)=5(y-1)$ $(10)$

Giải

Ta có $(10) \Leftrightarrow x^{2}(x-y)=5(y-x)+5(x-1) \Leftrightarrow\left(x^{2}+5\right)(x-y)=5(x-1) .$
Suy ra $5(x-1)\,  \vdots\, \left(x^{2}+5\right) \Rightarrow 5\left(x^{2}+5\right)-5 x(x-1)-5(x-1)\,  \vdots\, \left(x^{2}+5\right)$ hay $30\,  \vdots\,\left(x^{2}+5\right)$
$\Rightarrow\left(x^{2}+5\right) \in\{5 ; 6 ; 10 ; 15 ; 30\}$ và $x$ là số nguyên
$\Rightarrow x \in\{0 ; \pm 1 ; \pm 5\}$.

Thử lại ta được nghiệm nguyên của phương trình là $(0 ; 1); (1 ; 1); (-5 ;-4)$.

Ví dụ 11: Chứng minh phương trình: $x^2 -2^y =2015$ $(11)$ không có nghiệm nguyên.

Giải

$(11) \Leftrightarrow x^{2}=2015+2^y$.
Ta sẽ chứng minh $A=2015+2^{y}$ không phải là số chinh phương với mọi số nguyên $y$.

Thật vậy thay $y$ bằng $0 ; 1; 2$ thì $A$ lần lượt nhận các giá trị là $2016 ; 2017; 2019$ đều không phải là số chính phương. Với $y \geq 3$ thi $2^{y}$ chia hết cho $8$ , còn $2015$ chia $8$ dư $7$ nên $A$ chia $8$ dư $7$ mà số chính phương lẻ chia $8$ chỉ có thể dư $1$ do đó $A$ không phải là số chính phương.

Vậy phương trình  $(11)$ không có nghiệm nguyên.

Ví dụ 12: Tìm các số nguyên dương $a$, $b$ sao cho

$\dfrac{4}{a}+\sqrt[3]{4-b}=\sqrt[3]{4+4 \sqrt{b+b}}+\sqrt[3]{4-4 \sqrt{b}+b}$ $(12)$

Giải

Đặt $\sqrt[3]{2+\sqrt{b}}=x ; \sqrt[3]{2-\sqrt{b}}=y$.

Vì $b>0$ nên $x>0$. Ta có $xy = \sqrt[3]{2+\sqrt{b}} \cdot \sqrt[3]{2-\sqrt{b}}=\sqrt[3]{4-b}$; $x^3 + y^3 =4$

Do đó phương trình $(12)$ trở thành:
$\dfrac{x^{3}+y^{3}}{a}+x y=x^{2}+y^{2} \Leftrightarrow \dfrac{x^{2}+y^{3}}{a}=x^{2}+y^{2}-x y$
$\Leftrightarrow \dfrac{(x+y)\left(x^{2}+y^{2}-x y\right)}{a}=x^{2}+y^{2}-x y$
mà $x^{2}+y^{2}-x y=\dfrac{3}{4} x^{2}+\left(\dfrac{x}{2}-y\right)^{2}>0$

suy ra $x+y=a \Rightarrow \sqrt[3]{2+\sqrt{b}}+\sqrt[3]{2-\sqrt{b}}=a$ $(*)$
$\Rightarrow 4+3 \sqrt[3]{4-b} \cdot a=a^{3}$ $(**)$
$\Leftrightarrow 4-b=\left(\dfrac{a^{3}-4}{3 a}\right)^{3}$
Vì $b$ là số nguyên nên $a^{3}-4\, \vdots \, 3 a \Rightarrow a^{3}-4 \, \vdots \, a$ $\Rightarrow 4 \vdots a \Rightarrow a \in\{1 ; 2 ; 4\}$
Với $a=1 \Rightarrow b=5$.

Với $a=2$ hoặc $a=4$ thì $b$ không phải là số nguyên dưong.

Thử lại: Với $a=1$; $b=5$ ta có $(**)$ đúng tức là
$x^{3}+y^{3}+3 x y a=a^{3} \Leftrightarrow a^{3}-x^{3}-y^{3}-3 x y a=0$
$\Leftrightarrow(a-x-y)\left[(a+x)^{2}+(a+y)^{2}+(x-y)^{2}\right]=0 .$
Do $x>0 ; a>0$ nên $x+a>0 \Rightarrow(a+x)^{2}+(a+y)^{2}+(x-y)^{2}>0 \Rightarrow a-x-y=0$ hay $a=x+y$,
tức là $(*)$ đúng.

Vậy $(a, b)=(1 ; 5)$ là cặp số nguyên dương duy nhất thỏa mãn phương trình $(12)$.

Ví dụ 13: Tìm số tự nhiên $n$ thỏa mãn $361\left(n^{3}+5 n+1\right)=85\left(n^{4}+6 n^{2}+n+5\right)$  $(13)$

Giải
Ta có $(13) \Leftrightarrow \dfrac{n^{3}+5 n+1}{n^{4}+6 n^{2}+n+5}=\dfrac{85}{361}$.
Ta sẽ chứng minh $\dfrac{n^{3}+5 n+1}{n^{4}+6 n^{2}+n+5}$ la phân số tối giản với mọi giá trị $n \in \mathbb{N}$.
Thật vậy, đặt $d=\left(n^{3}+5 n+1 ; n^{4}+6 n^{2}+n+5\right)$.
Suy ra $n^{4}+6 n^{2}+n+5-n\left(n^{3}+5 n+1\right)\,  \vdots \, d$ hay $n^{2}+5\, \vdots\,  d$.

Từ đó $\left(n^{3}+5 n+1\right)-n\left(n^{2}+5\right)$ \,  \vdots \, d hay $1\,\vdots \, d \Rightarrow d=1$.

Vậy phân số $\dfrac{n^{3}+5 n+1}{n^{4}+6 n^{2}+n+5}$ là phân số tối giản.

Mặt khác phân số $\dfrac{85}{361}$ cũng là phân số tối giản mà dạng tối giản của một phân số là duy nhất nên ta có

$\left\{\begin{array}{l}n^{3}+5 n+1=85 \\ n^{4}+6 n^{2}+n+5=361\end{array}\right.$
$\Rightarrow\left(n^{4}+6 n^{2}+n+5\right)-n\left(n^{3}+5 n+1\right)=361-85 n$
$\Leftrightarrow n^{2}+85 n-356=0 \Leftrightarrow(n-4)(n+89)=0$
Vi $n \in \mathbb{N}$ nên $n=4$.

Vậy số tự nhiên cần tìm là $n=4$.

Ví dụ 14: Tìm tất cả các số nguyên dương $m$, $n$ thỏa mãn $3^{m}=n^{2}+2 n-8$ $(14)$

Giải
Ta có $(14) \Leftrightarrow 3^{m}=(n+4)(n-2)$.

Đặt $n+4=3^{x} ; n-2=3^y$ với $x, y$ là số tự nhiên và $x+y=m$, khi đó $3^{x}-3^{y}=6$ hay $3^{y}\left(3^{x-y}-1\right)=6$.

Vì $3^y$ chỉ có ước là lũy thừa của $3$; $3^{x-y}-1$ không chia hết cho $3$ và $6=3.2$ nên $3^{y}=3$ và $3^{x-y}-1=2$ hay $y=1$ và $x=2$.

Từ đó $m=x+y=3$ và $n=3^{y}+2=5$.

Ví dụ 15: Tìm các số nguyên dương $x$, $y$ thỏa $ x^{2}-y !=2015$

Giải
Nếu $y>5$ thì $y !\, \vdots \, 9 \Rightarrow y !+2015$ chia $9$ dư $8$ mà $x^{2}$ chia $9$ chi có thể nhận các số dư là $0 ; 1 ; 4 ; 7$ nên trong trường hợp này không tồn tại nghiệm.

Xét $y$ lần lượt bằng $0 ; 1 ; 2 ; 3 ; 4 ; 5$ đều không có giá trị $x$ thỏa mãn.
Vậy phương trình $(15)$ vô nghiệm.

Ví dụ 16: Tìm tất cả các số tự nhiên $m;n$ để $P=3^{3m^2+6n-61}+4$ là số nguyên tố.
(Trích đề thi HSG TP. Hà Tĩnh, năm hoc $2015-2016$)

Giải
Nhận xét:  Để tìm các số tự nhiên $m, n$ sao cho $P$ là số nguyên tố thì ta có thể chứng minh $P$ chia hết cho
một số nguyên tố $n$ nào đó và khi đó $P=n$

Đặt $3m^2+6n-61=3k+2\ (k\in \mathbb{N})$.

Ta có $P=3^{3 k-2}+4=9.27^{k}+4$

Vì $27\equiv 1 (\bmod 13)$ nên $27^{k}\equiv 1 (\bmod 13)\Rightarrow 9.27^{k} \equiv 9 (\bmod 13) \Rightarrow 9.27^{k}+4 \equiv 13(\bmod 13)$
hay $P\, \vdots\, 13$, mà $P$ là số nguyên tố nên $P=13$, điều này xảy ra khi và chỉ khi $k=0 .$

Suy ra $3 m^{2}+6 n-61=2 \Leftrightarrow m^{2}+2 n=21$
Vì $m ; n$ là các số tự nhiên nên chỉ có 2 cặp số $(m ; n)$ thỏa mãn là $(1 ; 10)$ và $(3 ; 6)$.

Ví dụ 17: Tìm nghiệm nguyên của phương trình $x^{11}+y^{11}=11 z$ $(17)$

Giải
$(17)$ có nghiệm nguyên khi $x^{11}+y^{11}\, \vdots \, 11$.

Vì $11$ là số nguyên tố, theo định lý nhỏ Fermat ta có: $x^{11}- x\, \vdots\, 11$ và $y^{11}-y\, \vdots\, 11 .$

Ta viết $(17)$ dưới dạng: $\left(x^{11}-x\right)+\left(y^{11}-y\right)+(x+y)=11 z$ suy ra $x+y\,  \vdots\,  11$.

Đặt $x+y=11 k ; x=t$ $(k, t \in \mathbb{Z}) .$ Ta có công thức nghiệm: $x=t$, $y=11 k-t$ và $\left[t^{11}+(11 k-t)^{11}\right] \, \vdots\, 11$.

Bài tập rèn luyện.

Bài 1. Giải các phương trình sau trong tập nguyên dương:
a) $ 2x^2+3xy-2y^2=7 $.
b) $ x^3-xy=6x-5y-8 $
c) $ x^3-y^3=91 $.

Bài 2. Tìm các số nguyên $x$, $y$ sao cho:

a) $3^x-y^3=1$;
b) $1+x+x^2+x^3=2^y$;
c) $1+x+x^2+x^3=2003^y$.

Bài 3. Tìm các số nguyên tố $x$, $y$, $z$ thỏa mãn: $x^y+1=z$

Bài 4. Tìm các số nguyên dương $x, y,z$ thỏa $y$ nguyên tố và $y, 3$ không là ước của $z$ thỏa $x^3-y^3=z^2$.

Bài 5. Chứng tỏ rằng các phương trình sau không có nghiệm nguyên

a) $2x^2 +y^2 =1999$.

b) $7x^2 -5y^2 =3$.

c) $x^4 + y^4 + (x+y)^4=4004$.

Bài 6. Tìm nghiệm nguyên của mỗi phương trình sau:

a) $17x^2 +26y^2 = 846$.

b) $3x^2 -3xy =7x -y -21$.

c) $x^3 + 3367 =2^y$.

d) $2^x -3^y =7$.

e) $x! + y! =10z+9$.

f) $|x-y|+|y-z|+|z-x|=2017$.

g) $x^3 +y^3 +z^4 =2003$.

Bài 7. Tồn tại hay không $4$ số nguyên liên tiếp $a$, $b$, $c$, $d$ thỏa mãn $a^3 + b^2 +c+d=491$.

Bài 8. Cho đa thức $f(x)$ có các hệ số nguyên. Biết rằng $f(1)\cdot f(2)=45$. Chứng tỏ đa thức $f(x)$ không có nghiệm nguyên.

Bổ đề về số mũ đúng

BỔ ĐỀ VỀ SỐ MŨ ĐÚNG

(Thầy Nguyễn Ngọc Duy giáo viên trường PTNK TP Hồ Chí Minh)

Bổ đề số mũ đúng của một số nguyên là một hướng tiếp cận khá mới đối với các bài toán sơ cấp. Nó cung cấp một công cụ khá hữu hiệu để giải các phương trình Diophante hoặc các bài toán chia hết liên quan đến số mũ. Trong bài viết này tôi sẽ cố gắng mang đến một cái nhìn thật sơ cấp và tự nhiên đến vấn đề, trang bị thêm kiến thức và kĩ năng cho các các em học sinh để giải quyết các bài toán số học. Đặc biệt, ta sẽ dùng bổ đề số mũ đúng để giải quyết một số trường hợp đặc biệt của định lí lớn Fermat.

1. Kiến thức cần nhớ

Định nghĩa 1.1: Cho $\left( a,n \right)=1$. Kí hiệu cấp của a theo modulo n là $or{{d}_{n}}\left( a \right)$, là số nguyên dương d nhỏ nhất thỏa $a^d\equiv 1\, \left( \bmod n \right)$.

Tính chất 1.1: Nếu $x$ là số nguyên dương thỏa mãn $a^x \equiv 1\, \left( \bmod n \right)$ thì $or{{d}_{n}}\left( a \right)|x$.

Định nghĩa 1.2: Cho $p$ là số nguyên tố, $x$ là số nguyên bất kì, kí hiệu $v_p \left( x \right)=n$ nếu $x$ chia hết cho $p^n$ nhưng không chia hết cho $p^{n+1}$ .

Tính chất 1.2: Với $a,b$ là các số nguyên và $n$ là số nguyên dương thì:

  • $v_p \left( ab \right)=v_p \left( a \right)+v_p \left( b \right)$.
  • Nếu $p|a$ thì $v_p(a) >0.$
  • $v_p \left( a^n \right)=n v_p \left( a \right)$.
  • $v_p \left( a+b \right) \ge \min \left\{ v_p \left( a \right), v_p \left( b \right) \right\}$. Đẳng thức xảy ra chẳng hạn khi $v_p(a) \neq v_p(b).$
  • $v_p(\text{gcd}(a,b)) = \min(v_p(a), v_p(b))$ và $v_p(\text{lcm}(a,b)) = \max(v_p(a), v_p(b)).$

Định lý 1.1: Bổ đề số mũ đúng. Cho $p$ là số nguyên tố lẻ; $a,b$ không chia hết cho $p$

$i)$  Nếu $a-b$ chia hết cho p thì $v_p \left( a^n – b^n \right)=v_p \left( a-b \right)+v_p \left( n \right)$.

$ii)$  Nếu $a+b$ chia hết cho $p, n$ lẻ thì $v_p\left( a^n+b^n \right)=v_p\left( a+b \right)+v_p \left( n \right)$.

$iii)$  Nếu $a, b$ lẻ thì $v_2 \left( a^n – b^n \right)=v_2 \left( \dfrac{x^2 – y^2}{2} \right) + v_2 \left( n \right)$.

Chứng minh
  • Trước tiên, ta chứng minh: ${{v}_{p}}\left( {{a}^{p}}-{{b}^{p}} \right)={{v}_{p}}\left( a-b \right)+1$ $(*)$. Ta có:

$${{a}^{p}}-{{b}^{p}}=\left( a-b \right)\left( {{a}^{p-1}}+{{a}^{p-2}}b+…+a{{b}^{p-2}}+{{b}^{p-1}} \right).$$

Do $a\equiv b\left( \bmod p \right)$ nên ${{a}^{p-1}}+{{a}^{p-2}}b+…+a{{b}^{p-2}}+{{b}^{p-1}}\equiv p.{{a}^{p-1}}\equiv 0\left( \bmod p \right)$.

Suy ra : ${{a}^{p-1}}+{{a}^{p-2}}b+…+a{{b}^{p-2}}+{{b}^{p-1}}$ chia hết cho $p$  $(1)$.

Ta chứng minh tiếp $${{a}^{p-1}}+{{a}^{p-2}}b+…+a{{b}^{p-2}}+{{b}^{p-1}} \text{không chia hết cho } {{p}^{2}}. $$

Thật vậy, do $a\equiv b\left( \bmod p \right)$ nên $a=b+kp$ . Sử dụng khai triển nhị thức Newton ta có

$ {{a}^{p-1}}+{{a}^{p-2}}b+\cdots+{{b}^{p-1}}$

$\equiv \left[ \left( p-1 \right)kp{{b}^{p-2}}+{{b}^{p-1}} \right]+\left[ \left( p-2 \right)kp{{b}^{p-2}}+{{b}^{p-1}} \right]+  \cdots+\left[ kp{{b}^{p-2}}+{{b}^{p-1}} \right]+{{b}^{p-1}}\left( \bmod {{p}^{2}} \right) $

$\equiv \dfrac{p\left( p-1 \right)}{2}kp{{b}^{n-2}}+p.{{b}^{p-1}}$

$\equiv p{{b}^{p-1}}\left( \bmod {{p}^{2}} \right) $

Theo giả thiết thì $b$ không chia hết cho $p$ nên $p{{b}^{p-1}}$ không chia hết cho ${{p}^{2}}$. Do đó ${{a}^{p-1}}+{{a}^{p-2}}b+\cdots+a{{b}^{p-2}}+{{b}^{p-1}}$ cũng không chia hết cho ${{p}^{2}}$  $(2)$.

Từ $(1), (2)$ ta có: ${{v}_{p}}\left( {{a}^{p-1}}+{{a}^{p-2}}b+\cdots+a{{b}^{p-2}}+{{b}^{p-1}} \right)=1$.

Vậy ${{v}_{p}}\left( {{a}^{p}}-{{b}^{p}} \right)={{v}_{p}}\left( a-b \right)+1$.

  • Tương tự, ta cũng có: nếu m không chia hết cho p thì ${{v}_{p}}\left( {{a}^{m}}-{{b}^{m}} \right)={{v}_{p}}\left( a-b \right)$ $(**)$.

Ta quay lại định lí. Đặt ${{v}_{p}}\left( n \right)=k\Rightarrow n={{p}^{k}}.m$, với $\left( m,p \right)=1$.

Áp dụng $(*)$ và $(**)$ ta có:

${{v}_{p}}\left( {{a}^{n}}-{{b}^{n}} \right)  ={{v}_{p}}\left( {{\left( {{a}^{{{p}^{k-1}}.m}} \right)}^{p}}-{{\left( {{b}^{{{p}^{k-1}}.m}} \right)}^{p}} \right) $

$={{v}_{p}}\left( {{a}^{{{p}^{k-1}}.m}}-{{b}^{{{p}^{k-1}}.m}} \right)+1=\ldots={{v}_{p}}\left( {{a}^{m}}-{{b}^{m}} \right)+k $

$={{v}_{p}}\left( a-b \right)+{{v}_{p}}\left( n \right).$

Vậy ta đã chứng minh xong phần $i)$ của định lí.

Vì $n$ lẻ nên thay $b$ bởi $-b$ trong i. ta được ${{v}_{p}}\left( {{a}^{n}}+{{b}^{n}} \right)={{v}_{p}}\left( {{a}^{n}}-{{\left( -b \right)}^{n}} \right)={{v}_{p}}\left( a+b \right)+{{v}_{p}}\left( n \right)$

Vậy ta đã chứng minh xong phần $ii)$ của định lí. Tương tự cách làm trong $i)$ ta cũng có kết quả $iii)$.

Như vậy ta đã chứng minh xong bổ đề số mũ đúng. Sau đây ta sẽ sử dụng bổ đề để giải quyết một bài toán thú vị.

2. Các bài toán áp dụng

Bài toán Fermat lớn: Cho $n$ là số tự nhiên lớn hơn $2.$ Chứng minh rằng phương trình ${{a}^{n}}+{{b}^{n}}={{c}^{n}}$ không có nghiệm nguyên dương.

Bài Toán Fermat lớn là bài toán cực kì thú vị. Nó tồn tại gần bốn thế kỉ, kích thích biết bao nhà toán học thế giới. Bài toán cuối cùng được chứng minh bởi nhà toán học Andrew Wiles vào năm 1993. Và người ta nói rằng sẽ không có phương pháp sơ cấp nào có thể chứng minh bài toán trên. Bài báo sẽ đề cập một trường hợp đặc biệt của bài toán: số $c$ là số nguyên tố. Và chúng ta sẽ giải quyết thông qua bổ đề số mũ đúng.

Bài toán 1: Cho số nguyên lẻ $n>2$, $p$ là số nguyên tố. Chứng minh rằng phương trình $a^n + b^n = p^n$ không có nghiệm nguyên dương.

Giải

Không mất tính tổng quát, giả sử phương trình có nghiệm $a\ge b$ .

$1.$ Nếu $a=1\Rightarrow b=1$, thế vào phương trình suy ra vô lí.

$2.$ Nếu $a=2\Rightarrow b=1;2$.

  • Trường hợp $\left( a,b \right)=\left( 2,2 \right)\Rightarrow p=2$ (vô lí).
  • Trường hợp $\left( a,b \right)=\left( 2,1 \right)\Rightarrow p=3$ , thế vào phương trình ta được ${{3}^{n}}-{{2}^{n}}=1$ , cũng suy ra vô lí.

Vậy bắt buộc $a\ge 3$, mà ${{p}^{n}}>{{a}^{n}}\Rightarrow p>3$ , nên p là số nguyên tố lẻ. Do n lẻ, ta có : $${{p}^{n}}={{a}^{n}}+{{b}^{n}}=\left( a+b \right)\left( {{a}^{n-1}}-{{a}^{n-2}}b+\cdots-a{{b}^{n-2}}+{{b}^{n-1}} \right) $$

Suy ra $p|a+b$ (do $a+b>1$ ). Áp dụng bổ đề số mũ đúng cho $p$, ta có

$${{v}_{p}}\left( {{a}^{n-1}}-{{a}^{n-2}}b+\cdots-a{{b}^{n-2}}+{{b}^{n-1}} \right)={{v}_{p}}\left( {{a}^{n}}+{{b}^{n}} \right)-{{v}_{p}}\left( a+b \right)={{v}_{p}}\left( n \right) $$

Mà ${{a}^{n-1}}-{{a}^{n-2}}b+\cdots-a{{b}^{n-2}}+{{b}^{n-1}}$ là lũy thừa của $p$ nên ta có $$\left( {{a}^{n-1}}-{{a}^{n-2}}b+\cdots-a{{b}^{n-2}}+{{b}^{n-1}} \right)|n.$$

Do ${{a}^{n-1}}-{{a}^{n-2}}b+\cdots-a{{b}^{n-2}}+{{b}^{n-1}}=\frac{1}{2}\left[ {{a}^{n-1}}+{{a}^{n-3}}{{\left( a-b \right)}^{2}}+\cdots+{{b}^{n-3}}{{\left( a-b \right)}^{2}}+{{b}^{n-1}} \right]\ge \dfrac{1}{2}\left( {{a}^{n-1}}+{{b}^{n-1}} \right)$

Vì $a\ge 3$, $n\ge 3$ nên $\frac{1}{2}\left( {{a}^{n-1}}+{{b}^{n-1}} \right)>n$ nên không thể $$\left( {{a}^{n-1}}-{{a}^{n-2}}b+\cdots-a{{b}^{n-2}}+{{b}^{n-1}} \right)|n.$$

Vậy phương trình vô nghiệm khi $p$ là số nguyên tố.

Bài tập 2: Cho số nguyên $n>2$ có ước lẻ khác 1, $p$ là số nguyên tố. Chứng minh rằng phương trình ${{a}^{n}}+{{b}^{n}}={{p}^{n}}$ không có nghiệm nguyên dương.

Giải

Gọi $k>1$ là ước lẻ của $n$, giả sử $n=km$ . Đặt $x={{a}^{m}};y={{b}^{m}}$. Phương trình trên trở thành

$${{x}^{k}}+{{y}^{k}}={{p}^{n}}.$$

Không mất tính tổng quát, giả sử $x\ge y$ . Tương tự bài toán $1$ ta sẽ loại được các trường hợp tầm thường $x=1;x=2$ . Nên ta xét bài toán với trường hợp $x,p\ge 3.$ Do $k$ lẻ, ta có ${{p}^{n}}={{a}^{k}}+{{b}^{k}}=\left( a+b \right)\left( {{a}^{k-1}}-{{a}^{k-2}}b+\cdots-a{{b}^{k-2}}+{{b}^{k-1}} \right)$

Suy ra $p|b+a$. Áp dụng bổ đề số mũ đúng cho $p$ ta có

$${{v}_{p}}\left( {{a}^{k-1}}-{{a}^{k-2}}b+\cdots-a{{b}^{k-2}}+{{b}^{k-1}} \right)={{v}_{p}}\left( {{a}^{k}}+{{b}^{k}} \right)-{{v}_{p}}\left( a+b \right)={{v}_{p}}\left( k \right) $$

Mà ${{a}^{k-1}}-{{a}^{k-2}}b+ \cdots-a{{b}^{k-2}}+{{b}^{k-1}}$ là lũy thừa của $p$ nên ta có $$\left( {{a}^{k-1}}-{{a}^{k-2}}b+\cdots-a{{b}^{k-2}}+{{b}^{k-1}} \right) | k$$

Lập luận tương tự bài toán $1$ ta cũng suy ra vô lí. Vậy phương trình vô nghiệm .

Bài tập 3: Cho số nguyên $n={{2}^{k}},k>1$ , p là số nguyên tố. Chứng minh rằng phương trình ${{a}^{n}}+{{b}^{n}}={{p}^{n}}$ không có nghiệm nguyên dương.

Giải

Tương tự Bài toán $1$, ta loại được các trường hợp tầm thường nên ta chỉ xét đối với trường hợp $a,b$ có ít nhất một số lớn hơn $2$, khi đó $p>3$. Phương trình trở thành dạng

$${{x}^{4}}+{{y}^{4}}={{p}^{{{2}^{k}}}}$$

trong đó $x, y$ có ít nhất một số lớn hơn $2$ và $\left( x,y \right)=1$.

Do $p$ lẻ nên $x, y$ khác tính chẵn lẻ. Không mất tính tổng quát, giả sử $x$ lẻ, $y$ chẵn. Ta có

$${{y}^{4}}={{p}^{{{2}^{k}}}}-{{x}^{4}}=\left( {{p}^{{{2}^{k-1}}}}+{{x}^{2}} \right)\left( {{p}^{{{2}^{k-1}}}}-{{x}^{2}} \right)$$

Do $\left( {{p}^{{{2}^{k-1}}}}+{{x}^{2}};{{p}^{{{2}^{k-1}}}}-{{x}^{2}} \right)=2$ nên

$$\left\{ \begin{array}{l} {{p}^{{{2}^{k-1}}}}+{{x}^{2}}=2{{m}_{1}}^{2} \\ {{p}^{{{2}^{k-1}}}}-{{x}^{2}}=2{{n}_{1}}^{2} \end{array} \right. $$

Suy ra

$$\left\{ \begin{array}{l} {{p}^{{{2}^{k-1}}}}={{m}_{1}}^{2}+{{n}_{1}}^{2} \\ {{x}^{2}}={{m}_{1}}^{2}-{{n}_{1}}^{2} \end{array} \right. $$

và ${{y}^{2}}=2{{m}_{1}}{{n}_{1}}.$

Ta thấy $\left( {{m}_{1}};{{n}_{1}} \right)=1$ vì nếu ngược lại thì ${{m}_{1}}$ và ${{m}_{2}}$ đều phải chia hết cho $p$ (vô lí) nên có các trường hợp sau

$1)$ Nếu $m_1 = m_2^2, n_1=2n_2^2$ và $(m_2,n_2)=1$ thì thế vào ta được

$${{p}^{{{2}^{k-1}}}}={{m}_{2}}^{4}+4{{n}_{2}}^{4}=\left( {{m}_{2}}^{2}+2{{m}_{2}}{{n}_{2}}+2{{n}_{2}}^{2} \right)\left( {{m}_{2}}^{2}-2{{m}_{2}}{{n}_{2}}+2{{n}_{2}}^{2} \right)$$

mà \[\left( {{m}_{2}}^{2}+2{{m}_{2}}{{n}_{2}}+2{{n}_{2}}^{2},{{m}_{2}}^{2}-2{{m}_{2}}{{n}_{2}}+2{{n}_{2}}^{2} \right)=1\] nên \[{{m}_{2}}^{2}-2{{m}_{2}}{{n}_{2}}+2{{n}_{2}}^{2}=1\Leftrightarrow {{\left( {{m}_{2}}-{{n}_{2}} \right)}^{2}}+{{n}_{2}}^{2}=1\Leftrightarrow {{m}_{2}}={{n}_{2}}=1.\] Trường hợp này không thỏa.

$2)$ Nếu $m_1=2m_2^2,n_1=n_2^2$ và $(m_2,n_2)=1$ thì cũng tương tự.

Vậy phương trình không có nghiệm nguyên dương.

Như vậy sử dụng bổ đề số mũ đúng ta đã chứng minh được một trường hợp đặc biệt của Định lí lớn Fermat.

Sau đây, chúng ta sẽ sử dụng Bổ đề số mũ đúng để giải quyết một số bài toán khác.

Bài tập 4: Tìm bộ số nguyên dương $\left( a,b,p \right)$ trong đó $p$ là số nguyên tố thỏa $${{2}^{a}}+{{p}^{b}}={{15}^{a}}.$$

Giải

Ta có $\forall x,y\in \mathbb{Z};n\in \mathbb{N}$ thì ${{x}^{n}}-{{y}^{n}}\vdots x+y$ nên ${{p}^{b}}={{15}^{a}}-{{2}^{a}}\vdots 13\Rightarrow p=13.$

Áp dụng bổ đề

$$b={{v}_{13}}\left( {{13}^{b}} \right)={{v}_{13}}\left( {{15}^{a}}-{{2}^{a}} \right)={{v}_{13}}\left( 15-2 \right)+{{v}_{13}}\left( a \right)\Rightarrow {{v}_{13}}\left( a \right)=b-1\Rightarrow a \ \vdots \  {{13}^{b-1}}$$

Mà $a>0$ nên $a\ge {{13}^{b-1}}$, suy ra

${{13}^{b}}  ={{15}^{a}}-{{2}^{a}}=\left( 15-2 \right)\left( {{15}^{a-1}}+{{15}^{a-2}}.2+\cdots +{{15.2}^{a-2}}+{{2}^{a-1}} \right) $

$ \ge \left( 15-2 \right)\left( {{15}^{{{13}^{b-1}}-1}}+{{15}^{{{13}^{b-1}}-2}}.2+\cdots+{{15.2}^{{{13}^{b-1}}-2}}+{{2}^{{{13}^{b-1}}-1}} \right) $

$\Rightarrow b=1\Rightarrow a=1.$

Vậy nghiệm bài toán là $\left( a,b,p \right)=\left( 1,1,13 \right)$.

 

Bài tập 5: Chứng minh rằng không tồn tại cặp số $\left( a,n \right)$ nguyên dương, $n>2$ , sao cho ${{\left( a+1 \right)}^{n}}-{{a}^{n}}$ là lũy thừa bậc dương của $5.$

Giải

Giả sử tồn tại số nguyên dương $m$ sao cho $${{\left( a+1 \right)}^{n}}-{{a}^{n}}={{5}^{m}}.$$

Nhận xét: nếu$a$ hoặc $a+1$ chia hết cho $5$ thì số còn lại cũng cũng chia hết cho $5$ (vô lí). Nên cả hai số đều không chia hết cho $5.$ Ta xét các trường hợp:

$1.$  Nếu $a\equiv 1\left( \bmod 5 \right)\Rightarrow 0\equiv {{\left( a+1 \right)}^{n}}-{{a}^{n}}\equiv {{2}^{n}}-1\left( \bmod 5 \right)$ . Suy ra $4|n$.

$2.$  Nếu $a\equiv 2\left( \bmod 5 \right)\Rightarrow 0\equiv {{\left( a+1 \right)}^{n}}-{{a}^{n}}\equiv {{3}^{n}}-{{2}^{n}}\left( \bmod 5 \right)$. Suy ra $2|n$.

$3.$  Nếu $a\equiv 3\left( \bmod 5 \right)\Rightarrow 0\equiv {{\left( a+1 \right)}^{n}}-{{a}^{n}}\equiv {{4}^{n}}-{{3}^{n}}\left( \bmod 5 \right)$. Suy ra $4|n$.

Do đó, $n$ luôn là số chẵn, đặt $n=2{{n}_{1}}$, $\left( {{n}_{1}}\in \mathbb{N},{{n}_{1}}\ge 2 \right)$. Ta có

$ {{5}^{m}} = {{\left( a+1 \right)}^{2{{n}_{1}}}}-{{a}^{2{{n}_{1}}}}=\left( {{\left( a+1 \right)}^{2}}-{{a}^{2}} \right)\left( {{\left( a+1 \right)}^{2\left( {{n}_{1}}-1 \right)}}+ \cdots + {{\left( a+1 \right)}^{2}}{{a}^{2\left( {{n}_{1}}-2 \right)}}+{{a}^{2\left( {{n}_{1}}-1 \right)}} \right) $

$=\left( 2a+1 \right)\left( {{\left( a+1 \right)}^{2\left( {{n}_{1}}-1 \right)}}+{{\left( a+1 \right)}^{2\left( {{n}_{1}}-2 \right)}}{{a}^{2}}+…+{{\left( a+1 \right)}^{2}}{{a}^{2\left( {{n}_{1}}-2 \right)}}+{{a}^{2\left( {{n}_{1}}-1 \right)}} \right). $

Suy ra $5| 2a+15$ , áp dụng bổ đề số mũ đúng ta được

${{v}_{5}}\left( {{\left( a+1 \right)}^{2\left( {{n}_{1}}-1 \right)}}+{{\left( a+1 \right)}^{2\left( {{n}_{1}}-2 \right)}}{{a}^{2}}+…+{{\left( a+1 \right)}^{2}}{{a}^{2\left( {{n}_{1}}-2 \right)}}+{{a}^{2\left( {{n}_{1}}-1 \right)}} \right) $

$= {{v}_{5}}\left( {{\left( a+1 \right)}^{2{{n}_{1}}}}-{{a}^{2{{n}_{1}}}} \right)-{{v}_{5}}\left( 2a+1 \right)={{v}_{5}}\left( {{n}_{1}} \right). $

Do ${{\left( a+1 \right)}^{2\left( {{n}_{1}}-1 \right)}}+{{\left( a+1 \right)}^{2\left( {{n}_{1}}-2 \right)}}{{a}^{2}}+ \cdots +{{\left( a+1 \right)}^{2}}{{a}^{2\left( {{n}_{1}}-2 \right)}}+{{a}^{2\left( {{n}_{1}}-1 \right)}}$ là lũy thừa của $5$ nên $${{n}_{1}}\vdots \left( {{\left( a+1 \right)}^{2\left( {{n}_{1}}-1 \right)}}+{{\left( a+1 \right)}^{2\left( {{n}_{1}}-2 \right)}}{{a}^{2}}+…+{{\left( a+1 \right)}^{2}}{{a}^{2\left( {{n}_{1}}-2 \right)}}+{{a}^{2\left( {{n}_{1}}-1 \right)}} \right)$$ (vô lí vì về phải gồm ${{n}_{1}}$ số nguyên dương, ${{n}_{1}}>1$ và $a+1\ge 2$).

Vậy không tồn tại cặp số $\left( a,n \right)$ nguyên dương, $n>2$ sao cho ${{\left( a+1 \right)}^{n}}-{{a}^{n}}$ là lũy thừa bậc dương của $5.$

 

Bài tập 6: Cho hai số nguyên $a,n\ge 2$ sao cho tồn tại số nguyên dương k thỏa $n|{{\left( a-1 \right)}^{k}}$ . Chứng minh rằng n là ước của $1+a+{{a}^{2}}+…+{{a}^{n-1}}$ .

Giải

Giả sử $p$ là ước nguyên tố bất kì của $n$ . Theo giả thiết $n|{{\left( a-1 \right)}^{k}}$ nên p cũng là ước của $a-1$ .

Do ${{a}^{n}}-1=\left( a-1 \right)\left( 1+a+{{a}^{2}}+\cdots +{{a}^{n-1}} \right)$ nên áp dụng bổ đề số mũ đúng ta có

$${{v}_{p}}\left( 1+a+{{a}^{2}}+\cdots+{{a}^{n-1}} \right)={{v}_{p}}\left( {{a}^{n}}-1 \right)-{{v}_{p}}\left( a-1 \right)={{v}_{p}}\left( n \right).$$

Do mọi ước nguyên tố $p$ của n đều thỏa điều trên nên ta có $$n|1+a+{{a}^{2}}+\cdots+{{a}^{n-1}}.$$

Bài tập 7 (HSG Trung Quốc 2009): Tìm cặp số nguyên tố $\left( p,q \right)$ thỏa $pq|{{5}^{p}}+{{5}^{q}}$ (*).

Giải

Ta xét các trường hợp

$1.$   $p=q=5$ thỏa mãn bài toán.

$2.$   Nếu có một số bằng $5$, một số khác $5$. Không mất tính tổng quát, giả sử $p=5;q\ne 5$. Ta có :

$$5q|{{5}^{5}}+{{5}^{q}}\Leftrightarrow q|{{5}^{4}}+{{5}^{q-1}}\Leftrightarrow q|{{5}^{4}}+1=626$$ do ${{5}^{q-1}}\equiv 1\left( \bmod \,q \right)$ nên suy ra $q=2$ hoặc $q=313$.

$3.$  Nếu cả hai số $p,q\ne 5$ . Do ${{5}^{p}}\equiv 5\left( \bmod p \right),\,\,{{5}^{q}}\equiv 5\,\,\,\,\left( \bmod \,q \right)$ nên

$$\left( * \right)\Leftrightarrow \left\{ \begin{array}{l}  {{5}^{p-1}}+1\vdots q \\ {{5}^{q-1}}+1\vdots p \end{array} \right. \Rightarrow \left\{ \begin{array}{l} {{5}^{2\left( p-1 \right)}}-1\vdots q \\ {{5}^{2\left( q-1 \right)}}-1\vdots p \end{array} \right.$$

Do ${{5}^{2\left( p-1 \right)}}-1$ chia hết cho $q$ nhưng ${{5}^{p-1}}-1$ không chia hết cho $q$ nên

$${{v}_{2}}\left( \text{ord}_{q}\left( 5 \right) \right)=1+{{v}_{2}}\left( p-1 \right) .$$

Do ${{5}^{q-1}}-1$ chia hết $q$ nên $q-1\vdots or{{d}_{q}}\left( 5 \right)$ nên

$${{v}_{2}}\left( q-1 \right)\ge 1+{{v}_{2}}\left( p-1 \right) .$$

Tương tự khi xét chia hết cho $p$ ta lại có ${{v}_{2}}\left( p-1 \right)\ge 1+{{v}_{2}}\left( q-1 \right)$ (vô lí).

Vậy các cặp số thỏa mãn là $\left( p,q \right)=\left( 2,5 \right);\left( 5,2 \right);\left( 5,5 \right);\left( 5,313 \right);\left( 313,5 \right).$

Bài tập 8 (HSG Brazil 2009): Cho hai số nguyên tố $p, q$ sao cho $q=2p+1$ . Chứng minh rằng tồn tại một số là bội của $q$ có tổng các chữ số của nó trong hệ cơ số $10$ nhỏ hơn $4.$

Giải

Do $p,q$ đều là số nguyên tố nên $q\ge 5$ .

Nếu $q=5$ thì ta chỉ cần chọn số $10$ thì thỏa yêu cầu bài toán.

Nếu $q>5$ , áp dụng Định lí Fermat nhỏ thì $q|{{10}^{q-1}}-1={{10}^{2p}}-1=\left( {{10}^{p}}-1 \right)\left( {{10}^{p}}+1 \right)$

Suy ra $q|{{10}^{p}}+1$ hoặc $q|{{10}^{p}}-1$.

$1.$  Nếu $q|{{10}^{p}}+1$ thì số $a={{10}^{p}}+1$ là số thỏa yêu cầu đề bài.

$2.$  Nếu $q|{{10}^{p}}-1$. Do $p$ là số nguyên tố và $q$ không là ước của $10-1$(do $q>5$ ) nên $p$ cũng chính là $or{{d}_{q}}\left( 10 \right)$. Do đó $10;{{10}^{2}};\ldots ;{{10}^{p}}$ sẽ có số dư khác nhau khi chia cho $q.$

Ta sẽ có các trường hợp

  • Nếu tồn tại $1\le k\le p$ mà ${{10}^{k}}\equiv p\left( \bmod \,q \right)$ thì ${{2.10}^{k}}+1\equiv 2p+1\equiv 0\left( \bmod \,q \right)$. Khi đó số $a={{2.10}^{k}}+1$ là số thỏa yêu cầu đề bài.
  • Nếu tồn tại $1\le k\le p$ mà ${{10}^{k}}\equiv 2p\left( \bmod \,q \right)$ thì ${{10}^{k}}+1\equiv 2p+1\equiv 0\left( \bmod \,q \right)$. Khi đó số $a={{10}^{k}}+1$ là số thỏa yêu cầu đề bài.
  • Nếu không tồn tại $1\le k\le p$ mà ${{10}^{k}}$ có số dư là $p$ hay $2p$ khi chia cho $q.$ Thì ta sẽ chia các số dư còn lại của $q$ thành $p$ bộ $$\left( 1;2p-1 \right),\left( 2;2p-2 \right),\ldots,\left( p-1;p+1 \right)$$ (tổng $2$ phần tử của một bộ bằng $2p$) . Do tập số dư khi chia cho $q$ của tập $\left\{ 10;{{10}^{2}};\ldots ;{{10}^{p}} \right\}$ có $p$ phần tử nên Theo nguyên lí Dirichlet sẽ có ít nhất hai số ${{10}^{k}}$ và ${{10}^{l}}$ thuộc cùng một bộ. Khi đó số $a={{10}^{k}}+{{10}^{l}}+1$ sẽ chia hết cho $q$ là số thỏa yêu cầu đề bài.

Bài tập 9 (IMO Shortlist 1997): Cho $b,m,n$ là các số nguyên dương thỏa$m>1;\,\,m\ne n$. Biết ${{b}^{m}}-1$và ${{b}^{n}}-1$ có cùng tập hợp các ước nguyên tố. Chứng minh $b+1$ là lũy thừa của $2.$

Giải

Theo đề, gọi $p$ là ước nguyên tố bất kì của ${{b}^{m}}-1$và ${{b}^{n}}-1$.

Ta có kết quả quen thuộc: $$\left( {{b}^{m}}-1,{{b}^{n}}-1 \right)={{b}^{\left( m,n \right)}}-1,$$ đặt $\alpha =\left( m,n \right)$ nên $p|{{b}^{\alpha }}-1$. Suy ra tồn tại $k,l\in \mathbb{N}*$ thỏa $m=\alpha k;\,\,n=\alpha l$.

Đặt $a={{b}^{\alpha }}$ , từ giả thiết suy ra mọi ước nguyên tố của ${{a}^{k}}-1$ và ${{a}^{l}}-1$ đều là ước của $a-1$ . Nói cách khác, tập hợp các ước nguyên tố của ${{a}^{k}}-1,{{a}^{l}}-1$ và $a-1$ là trùng nhau.

Do $m\ne n$ suy ra tồn tại một số $k$ hoặc $l$ lớn hơn 1. Giả sử số đó là k.

Ta chứng minh $a+1$ là lũy thừa của 2.

Thật vậy:

$1.$  Nếu $k$ là số chẵn, đặt $k={{2}^{\beta }}.k’$($k’$ là số lẻ).

Ta có: $${{a}^{k}}-1=\left( {{a}^{k’}}-1 \right)\left( {{a}^{k’}}+1 \right)\left( {{a}^{2k’}}+1 \right)…\left( {{a}^{{{2}^{\beta -1}}k’}}+1 \right).$$

Do đó mọi ước nguyên tố $q$ của ${{a}^{k’}}+1$ cũng là ước của $a-1$

Mà ${{a}^{k’}}+1\vdots a+1$, $\left( a+1;a-1 \right)=1$ hoặc $2.$ Suy ra $2\vdots q\Rightarrow q=2$ nên ${{a}^{k’}}+1$ là lũy thừa của $2.$ Suy ra $a+1$ cũng là lũy thừa của $2.$

$2.$  Nếu $k$ là số lẻ, ta có ${{a}^{k}}-1=\left( a-1 \right)\left( {{a}^{k-1}}+{{a}^{k-2}}+…+a+1 \right)$

Gọi $q$ là ước nguyên tố bất kì của ${{a}^{k-1}}+{{a}^{k-2}}+…+1$. Do ${{a}^{k-1}}+{{a}^{k-2}}+…+a+1$ là số lẻ nên, nên $q$ cũng lẻ và là ước của ${{a}^{k}}-1$ . Do đó q cũng là ước của $a-1$ .

Áp dụng bổ đề số mũ đúng của $q$ ta có

${{v}_{q}}\left( {{a}^{k-1}}+{{a}^{k-2}}+…+1 \right)={{v}_{q}}\left( {{a}^{k}}-1 \right)-{{v}_{q}}\left( a-1 \right)={{v}_{q}}\left( k \right)$

Suy ra $k\vdots \left( {{a}^{k-1}}+{{a}^{k-2}}+…+1 \right)$ (vô lí vì vế phải có k số nguyên dương, $a>1$ ).

Vậy $a+1={{b}^{\alpha }}+1$ là lũy thừa của $2$.

Vì ${{b}^{\alpha }}+1$ là lũy thừa của $2$ nên nếu $\alpha $ là số chẵn thì ${{b}^{\alpha }}+1={{\left( {{b}^{\alpha ‘}} \right)}^{2}}+1$ hoặc là số lẻ hoặc chia 4 dư 2 nên chỉ có một trường hợp thỏa là $b=1$ . Còn nếu $\alpha $ là số lẻ thì ${{b}^{\alpha }}+1=\left( b+1 \right)\left( {{b}^{\alpha -1}}+{{b}^{\alpha -2}}+…+b+1 \right)$ nên $b+1$ cũng là lũy thừa của $2$.

Bài tập 10 (IMO Shortlist 1999): Tìm các số nguyên dương $n,p$ trong đó p nguyên tố thỏa ${{n}^{p-1}}|{{\left( p-1 \right)}^{n}}+1$.

Giải

Ta xét các trường hợp sau

$1.$  Nếu $p=2\Rightarrow n|2\Rightarrow n=1;2$ (thỏa).

$2.$  Nếu $p>2$ , suy ra $p$ lẻ nên ${{\left( p-1 \right)}^{n}}+1$ lẻ $\Rightarrow n$ lẻ

Gọi $q$ là ước nguyên tố nhỏ nhất của n $\Rightarrow q|{{n}^{p-1}}|{{\left( p-1 \right)}^{n}}+1$ $\Rightarrow q|{{\left( p-1 \right)}^{2n}}-1$

Mà : $q|{{\left( p-1 \right)}^{q-1}}-1\Rightarrow q|{{\left( p-1 \right)}^{\left( 2n,q-1 \right)}}-1$

Do n lẻ và $q$ là ước nguyên tố nhỏ nhất của n nên $\left( 2n;q-1 \right)=2$ .

Suy ra $q|{{\left( p-1 \right)}^{2}}-1=\left( p-2 \right)p$ $\Rightarrow $ $q|p-2$ hoặc $q=p$. Ta lại có các trường hợp nhỏ

$(a)$  Nếu $q|p-2\Rightarrow 0\equiv {{\left( p-1 \right)}^{n}}+1\equiv 1+1\equiv 2\left( \bmod \,q \right)$ $\Rightarrow q=2$ (vô lí vì q lẻ)

$(b)$  Nếu $q=p$ . Áp dụng bổ đề số mũ đúng cơ số q ta có

$\left( p-1 \right){{v}_{p}}\left( n \right)={{v}_{p}}\left( {{n}^{p-1}} \right)\le {{v}_{p}}\left[ {{\left( p-1 \right)}^{n}}+1 \right]={{v}_{p}}\left( p-1+1 \right)+{{v}_{p}}\left( n \right)=1+{{v}_{p}}\left( n \right)$

Suy ra : $\left( p-2 \right){{v}_{p}}\left( n \right)\le 1\Rightarrow p=3$ và ${{v}_{p}}\left( n \right)=1.$

Đến đây, bài toán trở thành : Tìm n để ${{n}^{2}}|{{2}^{n}}+1$.

Nhận xét $n=1$ thỏa yêu cầu bài toán nên ta xét $n>1$. Suy ra $n$ là số lẻ, gọi $r$ là ước nguyên tố nhỏ nhất của $n$. Suy ra $r|{{2}^{n}}+1\,\,|{{2}^{2n}}-1$, mà $r|{{2}^{r-1}}-1$ nên suy ra $r|{{2}^{\left( 2n;r-1 \right)}}-1$.

Do $n$ là số lẻ và $r$ là ước nguyên tố nhỏ nhất của $n$ nên $\left( 2n;r-1 \right)=2$ nên $r=3$. Ta có đánh giá sau

$$2{{v}_{3}}\left( n \right)\le {{v}_{3}}\left( {{4}^{n}}-1 \right)={{v}_{3}}\left( 4-1 \right)+{{v}_{3}}\left( n \right)\Rightarrow {{v}_{3}}\left( n \right)\le 1\Rightarrow {{v}_{3}}\left( n \right)=1.$$ Suy ra $n=3.m$, $\left( m,n \right)=1$. Thế vào đề bài, ta được $${{m}^{2}}|{{8}^{m}}+1|{{8}^{2m}}-1.$$

Nếu $m>1$ , tương tự ta gọi $s$ là ước nguyên tố nhỏ nhất của $m.$ Suy ra $m$ là ước của ${{8}^{2}}-1=63$. Do đó $s=7$, điều này vô lí vì ${{8}^{m}}+1$ chia $7$ dư $2.$ Suy ra $m=1\Rightarrow n=3$.

Vậy $\left( n,p \right)=\left( 1,2 \right);\left( 2,2 \right);\left( 3;3 \right)$ .

Các bài toán tổ hợp trên dãy số

CÁC BÀI TOÁN TỔ HỢP TRÊN DÃY SỐ

Thầy Lê Phúc Lữ 

(Lớp Cao học Khoa học tự nhiên TP.HCM)

Trong bài viết nhỏ này, chúng ta sẽ cùng xét khía cạnh tổ hợp của dãy số nguyên; khi cần đếm số lượng dãy thỏa mãn một điều kiện cho trước nào đó. Các phương pháp thường gặp: truy hồi, xuống thang, cực hạn, phản chứng, …

1. Các bài toán chọn lọc

Bài tập 1.1: Tìm tất cả các bộ số nguyên dương $x_1,\ x_2,\ x_3,\ \ldots ,\ x_{2017}$ sao cho có thể đặt chúng lên vòng tròn theo thứ tự đó mà $6$ số liên tiếp bất kỳ đều có thể chia thành hai nhóm $3$ có tổng bằng nhau.

Giải

Dùng phương pháp xuống thang.

Ta có $x_i+x_{i+1}+x_{i+2}+x_{i+3}+x_{i+4}+x_{i+5} \equiv 0 \pmod{2}$ với mọi $i=1,2,3,\ldots ,2017$ nên $x_i \equiv x_{i+6}$ với mọi $i.$

Vì $(6,2017)=1$ nên suy ra tất cả các số có cùng tính chẵn lẻ. Ta xét phép biến đổi dãy số sau:

  • Nếu tất cả các số cùng chẵn thì thay bằng $y_i=\dfrac{x_i}{2}$.
  • Nếu tất cả các số cùng lẻ thì thay bằng $y_i=\dfrac{x_i+1}{2}$.

Dễ thấy dãy mới cũng thỏa và tổng $S=\sum\limits_{i=1}^{2017} a_i$ sẽ giảm ngặt nếu có một số nào đó trong dãy khác $1$; suy ra quá trình biến đổi sẽ dừng lại khi tất cả đều là $1$. Vì ta thu được một dãy toàn là $1$ nên dãy ban đầu có tất cả các số hạng bằng nhau.

Nhận xét: Bài toán trên có thể thay việc chia 2 nhóm thành $3,4,5,\ldots $ nhóm và vẫn giải được bằng cách tương tự. Ta xét các bài tương tự sau:

Bài tập 1.2 (APMO 2017): Bộ năm số nguyên là tốt nếu có thể đặt chúng là $a,b,c,d,e$ để $a-b+c-d+e=29.$ Tìm tất cả các bộ $2017$ số sao cho $5$ số liên tiếp bất kỳ trong chúng đều tốt.

Ở bài toán này, điểm khó là không biết các số đã cho có dương hay không; vì thể, đại lượng tổng ở trên không xét tiếp tục được.

Tuy nhiên, cách áp dụng vẫn tương tự như sau:

  • Trừ tất cả các số của bộ cho $29$, ta thu được điều kiện tốt trở thành $a-b+c-d+e=0.$
  • Tất cả các số đã cho cùng tính chẵn lẻ, và chính xác là cùng chẵn.
  • Xét đại lượng $S=\sum\limits_{i=1}^{2017}{\left| \frac{{{a}_{i}}}{2} \right|}$ thì thông qua phép chia 2, tổng này giảm ngặt. Từ đó suy ra tất cả các số này phải là $0$ và tất cả ban đầu phải là $29.$

Bài tập 1.3 (VMO 2014): Tìm tất cả các bộ số $2014$ số hữu tỷ không âm sao cho nếu bỏ đi bất kỳ số nào trong chúng thì các số còn lại có thể được chia thành $3$ nhóm rời nhau, mỗi nhóm có $671$ số sao cho tích các số trong mỗi nhóm là bằng nhau.

Bài này khó hơn vì: số hữu tỷ chứ không nguyên, tích chứ không phải tổng, … Ta lần lượt giải quyết điều đó như sau:

  • Quy đồng mẫu để đưa về số nguyên.
  • Xét số mũ của 1 ước nguyên tố để đưa về tổng.
  • Chú ý thêm trường hợp số 0 (nếu có 1 số thì phải có ít nhất 4 số).

Bài tập 1.4: Cho dãy số nguyên dương $({{a}_{n}})$ thỏa mãn:

$i)$ Gồm các số phân biệt nhau.

$ii)$ Với mọi $n$ thì ${{a}_{n}}\ge n.$

$iii)$ $a_1=5,\ a_2=4,\ a_3=3$.

a) Chứng minh rằng tồn tại $n>2017$ sao cho $a_n \ne n+1$?

b) Giả sử $a_n=n+2$ với mọi $n>2017$, hỏi có tất cả bao nhiêu dãy số như thế?

Giải

a) Bài toán có thể giải quyết dễ dàng bằng phản chứng và Dirichlet. Thật vậy, nếu ${{a}_{n}}=n+1$ với mọi $n>2017$ thì các số hạng ${{a}_{4}}\to {{a}_{2017}}$ sẽ nhận các giá trị trong tập hợp $6\to 2018$. Khi đó, sẽ có hai số hạng bằng nhau, không thỏa.

b) Nếu đã có ${{a}_{n}}=n+2$ với mọi $n>2017$ thì các số hạng ${{a}_{4}}\to {{a}_{2017}}$ sẽ nhận các giá trị trong tập hợp $6\to 2019.$ Nhận xét:

  • ${{a}_{2017}}\in \left\{ 2017,2018,2019 \right\}$ nên có $3$ cách chọn.
  • ${{a}_{2016}}\in \left\{ 2016,2017,2018,2019 \right\}$ nhưng vì ${{a}_{2017}}$ đã lấy một số nên cũng còn $3$ cách chọn.
  • Tương tự, đến ${{a}_{6}}$ vẫn có $3$ cách chọn. Còn lại ${{a}_{5}}$ có $2$ cách chọn và ${{a}_{4}}$ có $1$ cách chọn.

Theo nguyên lý nhân, ta có $2\cdot {{3}^{2012}}$ dãy thỏa mãn.

Bài tập 1.5: Xét lục giác $ABCDEF$ có độ dài cạnh là $1$ được điền các số như hình vẽ

Một con ếch xuất phát từ $A$ và nhảy đến các đỉnh sao cho mỗi bước nhảy đều có độ dài nguyên. Hành trình của ếch là dãy các tên đỉnh mà ếch đã nhảy qua; và hai hành trình được coi là khác nhau nếu ở một lần thứ $k$ nào đó, đỉnh mà ếch nhảy đến ở hai hành trình là khác nhau.

Gọi $m$ là số hành trình ếch nhảy sao cho tổng các số mà nó nhảy qua là $2017$. Chứng minh rằng $m$ không phải là số chính phương.

Giải

Ta thấy $ACE$ và $BDF$ là hai tam giác đều có cạnh là $\sqrt{3}$ nên mỗi lần, ếch sẽ nhảy từ tam giác đều này đến tam giác đều kia.

Chia nhóm:

  • $I=(A,C,E)$ tương ứng với các số $(0,0,1)$.
  • $II=(B,D,F)$ tương ứng với $(1,1,2)$.

Ta thấy $\left\{ x+y|x\in I,y\in II \right\}=\left\{ 1,1,1,1,2,2,2,2,3 \right\}$ chứng tỏ tổng các số trên hai bước nhảy liên tiếp của ếch sẽ nhận giá trị là $4$ số $1$, $4$ số $2$ và $1$ số $3.$ Nếu gọi ${{s}_{n}}$ là số hành trình của ếch có tổng là $n$ thông qua chẵn bước thì

$${{s}_{n}}=4{{s}_{n-1}}+4{{s}_{n-2}}+{{s}_{n-3}}.$$

Một cách tương tự, gọi ${{t}_{n}}$ là số hành trình của ếch có tổng là $n$ thông qua lẻ bước thì công thức truy hồi vẫn thế (chỉ khác ở các số hạng đầu).

Vì vậy nên nếu gọi ${{u}_{n}}={{s}_{n}}+{{t}_{n}}$ là số hành trình của ếch có tổng là $n$ thì

$${{u}_{n}}=4{{u}_{n-1}}+4{{u}_{n-2}}+{{u}_{n-3}} \text{ với } n\ge 3.$$

Ta có ${{u}_{0}}=1,{{u}_{1}}=6,{{u}_{2}}=28$ và từ công thức truy hồi thì $m={{u}_{2017}}\equiv {{u}_{1}}\equiv 2 \pmod{4}$ nên $m$ không thể là số chính phương, ta có đpcm.

Nhận xét: Bài toán có thể giải bằng cách gọi $6$ dãy truy hồi $a_n,\ b_n,\ c_n,\ d_n,\ e_n,\ f_n$ chỉ số hành trình của ếch có tổng là $n$ và kết thúc tại $A,B,C,D,E,F$. Tuy nhiên, cách tiếp cận đó khá phức tạp, đòi hỏi phải khai thác nhiều các liên hệ giữa các đường đi.

Một bài toán tương tự:

Bài tập 1.6 (Ả Rập TST 2017): Người ta đặt các số $1,2,3,4$ trên vòng tròn theo thứ tự đó. Một con kiến xuất phát từ số $1$ và ở mỗi bước, nó sẽ bò qua số bên cạnh. Hỏi con kiến có bao nhiêu cách bò sao cho tổng tất cả các số mà nó bò qua (kể cả số ban đầu) bằng 21?

Tương tự bài trên, ta cũng tìm được hệ thức truy hồi là $s_n=s_{n-3}+2s_{n-5}+s_{n-7}$. Từ đó tính được $s_{21}=167.$

Bài tập 1.7: Đếm số dãy số nguyên dương $\left( a_1,\ a_2,\ \ldots ,\ a_{12}\right) $ thỏa mãn các điều kiện sau:

a) $1\le a_1 \le a_2 \le \ldots \le a_{12} \le 2017$

b) $a_i \equiv i^2 (\bmod 12)$.

Giải

Theo giả thiết, ta có

${{a}_{1}}\equiv {{a}_{5}}\equiv {{a}_{7}}\equiv {{a}_{11}}\equiv 1\text{ }(\bmod 12) $

$ {{a}_{2}}\equiv {{a}_{4}}\equiv {{a}_{8}}\equiv {{a}_{10}}\equiv 4\text{ }(\bmod 12) $

$ {{a}_{3}}\equiv {{a}_{9}}\equiv 9\text{ }(\bmod 12) $

$ {{a}_{6}}\equiv {{a}_{12}}\equiv 0\text{ }(\bmod 12) $

Đặt ${{a}_{i}}=12{{b}_{i}}+{{r}_{i}}$ với $i=1,2,3,\ldots ,12$ và ${{r}_{i}}$ là số dư tương ứng đã chỉ ra ở trên.

Do tính không giảm của dãy nên ta phải có

$$0\le {{b}_{1}}\le {{b}_{2}}\le {{b}_{3}}<{{b}_{4}}<{{b}_{5}}<{{b}_{6}}\le {{b}_{7}}\le {{b}_{8}}\le {{b}_{9}}<{{b}_{10}}<{{b}_{11}}<{{b}_{12}}\le 168.$$

Từ đó suy ra

$0\le {{b}_{1}}<{{b}_{2}}+1<{{b}_{3}}+2<{{b}_{4}}+2<{{b}_{5}}+2<{{b}_{6}}+2\le {{b}_{7}}+3\le {{b}_{8}}+4\le {{b}_{9}}+5 $

$<{{b}_{10}}+5<{{b}_{11}}+5<{{b}_{12}}+5\le 173 $

Do các số liệt kê ở trên đều phân biệt và thuộc $[0;173]$ nên số cách chọn một bộ như thế là $C_{174}^{12}$. Đó cũng chính là số dãy cần tìm.

Nhận xét: Điều kiện thứ hai có thể thay bằng một hàm số tùy ý theo $i$ chứ không nhất thiết phải là ${{i}^{2}}$, cách giải vẫn tương tự như trên.

Bài tập 1.8: Hỏi có bao nhiêu hoán vị $a_1,\ a_2,\ …,\ a_{2017}$ của $2017$ số nguyên dương đầu tiên thỏa mãn đồng thời các điều kiện sau:

$i)$ $a_{i+1}-a_i\le 1$ với mọi $i=1,2,3,\ldots,2016.$

$ii)$ Có đúng một chỉ số $i$ với $1\le i\le 2017$ sao cho $a_i=i$?

Giải

Trước hết, ta sẽ chứng minh nhận xét rằng số hoán vị của $n$ số nguyên dương đầu tiên thỏa mãn điều kiện i), gọi là hoán vị đẹp, sẽ là ${{2}^{n-1}}$. Thật vậy,

  • Đầu tiên, ta đặt số $1$ vào hoán vị.
  • Số $2$ có thể xếp trước hoặc sau số $1$, có $2$ cách.
  • Số $3$ có thể xếp vào đầu dãy hoặc ngay sau số $2$ đã xếp trước đó, có $2$ cách.
  • Số $4$ có thể xếp vào đầu dãy hoặc ngay sau số $3$ đã xếp trước đó, cũng có $2$ cách. Cứ như thế cho đến $n.$

Do đó, có tất cả ${{2}^{n-1}}$ cách xếp, tương ứng vời ${{2}^{n-1}}$ hoán vị.

Tiếp theo, giả sử ta có ${{a}_{i}}=i$.

Khi đó ${{a}_{i+1}}\le {{a}_{i}}+1=i+1$, nhưng không thể có ${{a}_{i+1}}=i+1$ (do chỉ có 1 chỉ số thỏa mãn ii) nên ${{a}_{i+1}}\le i$, mà ${{a}_{i}}=i$ nên ${{a}_{i+1}}\le i-1$. Tiếp theo, ${{a}_{i+2}}\le {{a}_{i+1}}+1\le i$ nên ${{a}_{i+2}}\le i-1$.

Do đó, các số từ ${{a}_{i+1}}$ đến ${{a}_{2017}}$ nhận giá trị không vượt quá $i-1$.

Lập luận tương tự, các số từ ${{a}_{1}}$ đến ${{a}_{i-1}}$ phải nhận giá trị không nhỏ hơn $i+1.$

Do đó, hai đoạn hoán vị phía trước và phía sau ${{a}_{i}}$ phải có độ dài bằng nhau, tức là ${{a}_{1009}}=1009$ là số ở giữa.

Rõ ràng các hoán vị phía trước và phía sau $1009$ đều phải là hoán vị đẹp và được sắp xếp độc lập với nhau.

Vậy số hoán vị cần tìm là ${{\left( {{2}^{1007}} \right)}^{2}}={{2}^{2014}}.$

Nhận xét: Nếu đề đổi số $2017$ thành $2018$ thì sẽ tồn tại hai chỉ số $i$ như trên và chúng sẽ cách đều hai đầu $1$ và $2018$. Khi đó, đoạn ở giữa cũng sẽ cố định, tức là có $i<j$ để

${{a}_{k}}=k$ với mọi $k=i,i+1,\ldots ,j$ và $i+j=2019.$

Phần trước $i$ và phần sau $j$ sẽ đổi chỗ cho nhau với số cách xếp là ${{({{2}^{i-1}})}^{2}}$.

Bài tập 1.9: Cho dãy các số nguyên dương $(u_n)$ thỏa mãn điều kiện

$0\le u_{m+n}-u_m-u_n \le 1$ với mọi $m,n\in \mathbb{Z}^+$.

Chứng minh rằng tồn tại $a\in \mathbb{R}^+$ sao cho $-1\le u_n-\left[ an \right]\le 1$ với mọi $n=1,2,3,\ldots ,2017.$

Giải

Ta đưa điều cần chứng minh về

$$\frac{{{u}_{n}}}{n}<a<\frac{{{u}_{n}}+1}{n}.$$

Đến đây, gọi

$$m=\min \left\{ \left. \frac{{{u}_{n}}+1}{n} \right|n=1,2,3,\ldots ,2017 \right\}$$ và

$$M=\max \left\{ \left. \frac{{{u}_{n}}}{n} \right|n=1,2,3,\ldots ,2017. \right\}$$

Cần chỉ ra $m>M$ rồi chọn số $a$ nằm giữa $(m,M)$ là xong. Gọi $p,q$ lần lượt là các chỉ số nhỏ nhất để có dấu bằng xảy ra ở các đánh giá trên. Khi đó

${{u}_{p}}+1=pm$ và ${{u}_{q}}=Mq.$

Ngoài ra, ${{u}_{k}}+1>km,\forall k<p$ và ${{u}_{k}}<kq,\forall k<q.$

  • Nếu $p=q$ thì hiển nhiên đúng.
  • Nếu $p>q$, ta đặt $p=q+k$ thì $k<p$ nên ${{u}_{k}}+1>km$, vì ${{u}_{p}}\ge {{u}_{q}}+{{u}_{k}}$ (theo giả thiết) nên $pm-1>Mq+km-1\Leftrightarrow m>M.$
  • Nếu $p<q$ thì cũng chứng minh tương tự với chú ý rằng ${{u}_{q}}\le {{u}_{p}}+{{u}_{k}}+1.$

Nhận xét: Nếu đề bài đổi giả thiết thành $0\le u_{m+n}-u_m-u_n\le 2$,

ta sẽ cần đến hai số $a,b$ sao mới thỏa mãn được kết luận (vì khoảng chênh lệch của các số hạng rộng hơn một tí), cụ thể là tồn tại $a,b>0$ để

$$-1\le u_n-\left[ an \right]-\left[ bn \right]\le 1.$$

Ở bài toán trên, ta còn chứng minh được một kết quả mạnh hơn là tồn tại $a$ để $u_n=[an]$ với mọi $n.$ Một bài toán tương tự trong đề trường Đông miền Trung:

Bài tập 1.10: Cho hàm số $f:\mathbb{R}\to \mathbb{R}$ thỏa mãn $\left| f(x+y)-f(x)-f(y) \right|\le 1,\forall x,y\in \mathbb{R}$. Chứng minh rằng tồn tại hàm cộng tính $g:\mathbb{R}\to \mathbb{R}$ thỏa mãn $\left| f(x)-g(x) \right|\le 1,\forall x.$

Đây có thể nói là một phiên bản trên $\mathbb{R}$ của bài toán trên (thay vì xét trên $\mathbb{N}$).

Tiếp theo, ta xét lớp các bài toán sử dụng một định lý thú vị trong dãy số, số học. Trước hết, ta xét định lý Beatty với nội dung như sau:

Cho hai số vô tỷ dương $\alpha ,\beta $. Xét hai dãy số:

  • $[\alpha ],[2\alpha ],[3\alpha ],\ldots $ tạo thành dãy $A.$
  • $[\beta ],[2\beta ],[3\beta ],\ldots $ tạo thành dãy $B.$

Khi đó $\dfrac{1}{\alpha }+\dfrac{1}{\beta }=1$ khi và chỉ khi $A,B$ là phân hoạch của $\mathbb{Z}^+$.

Chứng minh

Định lý này có thể chứng minh bằng cách sử dụng các BĐT về phần nguyên. Dưới đây là cách chứng minh cho chiều đảo:

Với mỗi số nguyên dương $k$, gọi $m,n$ là các số nguyên dương thỏa mãn

$$[m\alpha ]\le k<[(m+1)\alpha ] \text{ và } [n\beta ]\le k<[(n+1)\beta ].$$

Đặt $A=\{[i\alpha ],1\le i\le m\}$ và $B=\{[j\beta ],1\le j\le n\}$ thì $\left| A \right|=m,\left| B \right|=n$ và $A,B$ là phân hoạch của tập hợp $\left\{ 1,2,3,\ldots ,k \right\}$ theo định nghĩa của đề bài.

Do đó $m+n=k$. Theo bất đẳng thức phần nguyên thì $m\alpha -1<k<(m+1)\alpha $ nên $\dfrac{m}{k+1}<\dfrac{1}{\alpha }<\dfrac{m+1}{k}$.

Tương tự $\dfrac{n}{k+1}<\dfrac{1}{\beta }<\dfrac{n+1}{k}.$

Suy ra $$\dfrac{m+n}{k+1}<\dfrac{1}{\alpha }+\dfrac{1}{\beta }<\dfrac{m+n+2}{k} \text{ hay } \dfrac{k}{k+1}<\dfrac{1}{\alpha}+\dfrac{1}{\beta }<\dfrac{k+2}{k}.$$

Cho $k\to +\infty $, ta thu được $\dfrac{1}{\alpha }+\dfrac{1}{\beta }=1.$

Bài tập 1.11: Hai dung dịch $A,B$ có đặc điểm: số đo thể tích của $1$ kg $A$ bằng số đo khối lượng của $1$ lít $B.$ Ngoài ra, $p$ lít $A$ nặng bằng $q$ lít $B$ với $p,q$ nguyên tố khác nhau. Mỗi dung dịch được chia cho vào các bình nhỏ giống nhau, cùng chứa $1$ lít và vỏ nặng $1$ kg. Chứng minh rằng có đúng một cách ghép các bình cùng loại ($A$ hoặc $B$) lại với nhau mà khối lượng của chúng thuộc khoảng $(2017;2018).$

Giải

Gọi $x,y$ lần lượt là khối lượng riêng của các dung dịch thì $\dfrac{1}{x}=1\cdot y,px=qy$ nên $x=\sqrt{\dfrac{q}{p}},y=\sqrt{\dfrac{p}{q}}.$

Khối lượng mỗi bình là $\alpha =1+\sqrt{\dfrac{q}{p}},\beta =1+\sqrt{\dfrac{p}{q}}$. Dễ thấy $\dfrac{1}{\alpha }+\dfrac{1}{\beta }=1$, thỏa mãn định lý Beatty.

Suy ra hai dãy $[m\alpha ],[n\beta ]$ là phân hoạch của số nguyên dương nên ta có đpcm.

Bài tập 1.12 (APMO 2006): Với mỗi số nguyên dương $n$, gọi $a_n,\ b_n$ lần lượt là số cách viết $10^n$ trong hệ nhị phân, ngũ phân. Chứng minh rằng $(a_n),(b_n)$ là phân hoạch của $\mathbb{Z}^+ \backslash \{1\}.$

Giải

Để giải bài này, chú ý rằng: số chữ số của $M$ trong hệ $p$ phân là $[{{\log }_{p}}M]+1$.

Ngoài ra, $\alpha ={{\log }_{2}}10,\beta ={{\log }_{5}}10$ thỏa mãn điều kiện của định lý Beatty.

Từ đó, ta có một nhận xét thú vị rằng: tổng số chữ số của ${{2}^{n}}$ và ${{5}^{n}}$ trong hệ thập phân là $n+1.$

Bài tập 1.13 (VN TST 2000): Cho số nguyên dương $k$. Dãy số $(u_n)$ xác định bởi: $u_1=1$ và $u_{n+1}$ là số nguyên dương nhỏ nhất không thuộc tập hợp

$$\left\{ u_1,\ u_2,\ \ldots ,\ u_n,\ u_1+k,\ u_2+2k,\ \ldots ,\ u_n+nk \right\}.$$

Chứng minh rằng tồn tại $\alpha $ vô tỷ dương sao cho $u_n=\left[ n\alpha \right]$ với mọi $n.$

Giải

Để giải bài toán này, ta xét đa thức $P(x)={{x}^{2}}+(k-2)x-k$ với $k$ là số nguyên dương đã cho thì $P(x)$ có hai nghiệm phân biệt trái dấu. Hơn nữa, ${{\Delta }_{P(x)}}={{(k-2)}^{2}}+4k={{k}^{2}}+4$, không thể là số chính phương với bất kì số k nguyên dương nào nên hai nghiệm này đều là số vô tỉ. Ta thấy $$P(1)=1+(k-2)-k=-1<0,P(2)=4+2(k-2)-k=k>0$$ nên nghiệm dương của phương trình $P(x)=0$ thuộc khoảng $(1,2)$. Gọi nghiệm đó là $a.$

Đặt $b=a+k$ thì $a,b$ đều vô tỉ và $ab=a(a+k)={{a}^{2}}+ak=2a+k=a+b$ nên $\dfrac{1}{a}+\dfrac{1}{b}=1$.

Xét $f(n)=[na],g(n)=[nb]=f(n)+kn$ với $n$ là số nguyên dương.

Ta sẽ chứng minh rằng ${{x}_{n}}=f(n)$ bằng quy nạp. Thật vậy,

– Với $n=1$, khẳng định hiển nhiên đúng vì $1<a<2.$

– Giả sử ${{x}_{n}}=f(n)$ với mọi $n=1,2,3,…,m$. Ta sẽ chứng minh rằng ${{x}_{m+1}}=f({{x}_{m+1}})$.

Ta có $f(i)={{x}_{i}},g(i)=f(i)+ik={{x}_{i}}+ik$ với mọi $i=1,2,3,…,m$ nên ta có tập hợp

$H=\left\{ {{x}_{1}},{{x}_{2}},…,{{x}_{m}},{{x}_{1}}+k,{{x}_{2}}+2k,…,{{x}_{m}}+mk \right\} $

$ =\left\{ f(1),f(2),…,f(m),g(1),g(2),…,g(m) \right\} $

Rõ ràng $f(m+1)\notin H$ và $g(n)>f(n)$ với mọi $n$, $f(n)$ là hàm số đồng biến trên $\mathbb{N}*$ nên ta thấy rằng $f(m+1)$ chính là số tự nhiên nhỏ nhất không thuộc H. Theo định nghĩa dãy số $({{x}_{n}})$ đã cho thì ta có ${{x}_{m+1}}=f(m+1)$.

Do đó, khẳng định cũng đúng với $m+1.$ Theo nguyên lí quy nạp, ta có đpcm. Vậy số tự nhiên cần tìm chính là $a$ là nghiệm dương của phương trình ${{x}^{2}}+(k-2)x-k=0$.

Nhận xét: Đây là một kết quả có từ $1959$. Ta có thể phân tích cách tiếp cận như sau:

Xuất phát từ việc $\alpha =\sqrt{2},\beta =\sqrt{2}+2$ thỏa mãn điều kiện Beatty. Ta có hai dãy với công thức

$a_n=\left[ n\sqrt{2} \right],\ b_n=a_n+2n$ là phân hoạch của $\mathbb{Z}^+$.

Từ đó, để giấu dãy $b_n$ đi, ta chỉ cần xét $a_n+2n$.

Để ý $a_1=1,\ a_2=2,\ a_3=4,\ b_1=3,\ b_2=6,\ b_3=10$ nên $a_4$ có thể định nghĩa là số nguyên dương nhỏ nhất không thuộc $\left\{ a_1,\ a_2,\ a_3,\ a_1+2,\ a_2+4,\ a_3+6 \right\}$. Đó chính là cơ sở để có bài toán trên.

Bài tập 1.14 (Dãy Wythoff): Cho chuỗi $S_1=1$. Chuỗi $S_n$ được tạo thành từ chuỗi $S_{n-1}$ bằng cách thay $1\to 01$ và $0\to 1.$ Các chuỗi $S_1,\ S_2,\ S_3,\ \ldots $ được ghép liên tiếp lại với nhau thành một chuỗi vô hạn $L$. Gọi $a_n$ là vị trí của số $1$ thứ $n$ trong chuỗi $L.$ Chứng minh rằng tồn tại $\alpha $ vô tỷ dương sao cho $a_n=\left[ n\alpha \right],\forall n.$

Ở đây, ta có nhận xét rằng số $0$ thứ $n$ được sinh ra bởi số $1$ thứ $n$ nên nếu gọi $k_n$ là số các số $0$ đứng trước số $1$ thứ $n$ và $b_n$ là vị trí của số $0$ thứ $n,$ ta sẽ có $a_n=n+k_n$ và $b_n=2n+k_n$ nên $b_n=a_n+n$.

Chú ý rằng $(a_n),\ (b_n)$ chính là phân hoạch của $\mathbb{Z}^+$ nên dễ dàng tìm được $\alpha $ là nghiệm của $\dfrac{1}{\alpha }+\dfrac{1}{\alpha +1}=1$ hay $\alpha $ chính là tỷ số vàng.

Bài tập 1.15: Cho $n$ là số nguyên dương, hỏi có bao nhiêu dãy số $a_1,\ a_2,\ \ldots ,\ a_{2n}$ sao cho

$i)$ $a_i \in \left\{ -1,1 \right\}$ với $i=1,2,3,\ldots ,2n.$

$ii)$ $\left| \sum\limits_{i=2k+1}^{2l} a_i \right|\le 2$ với $0\le k<l\le n$?

Giải

Gọi $S$ là tập hợp các dãy thỏa mãn đề bài và đặt $\left| S \right|={{s}_{n}}$. Gọi $T$ là tập hợp tất cả các tổng các ${{a}_{i}}$ lấy từ chỉ số lẻ bất kỳ đến $2n.$ Theo giả thiết thì $T\subset \left\{ \pm 2,\pm 1,0 \right\}$, tuy nhiên, tất cả các tổng trong $T$ đều có chẵn số hạng mà mỗi số hạng đều là $\pm 1$ nên tất cả phải đều chẵn. Suy ra $T\subset \left\{ \pm 2,0 \right\}$.

Nếu trong $T$ chứa cả $2$ lẫn $-2$ thì giả sử $\sum\limits_{k=2i+1}^{2n}{{{a}_{k}}}=2$ và $\sum\limits_{k=2j+1}^{2n}{{{a}_{k}}}=-2$ với $i<j$ , khi đó

\[4=\sum\limits_{k=2i+1}^{2n}{{{a}_{k}}}-\sum\limits_{k=2j+1}^{2n}{{{a}_{k}}}=\sum\limits_{k=2i+1}^{2j}{{{a}_{k}}},\] mâu thuẫn.

Ứng với $({{a}_{1}},{{a}_{2}},\ldots ,{{a}_{2n}})\in S$, ta có phân loại sau:

  • Tất cả các tổng trong $T$ đều là $0$, đặt số lượng dãy có tính chất này là ${{a}_{n}}$.
  • Trong $T$ có chứa số $2$, đặt số lượng dãy có tính chất này là ${{b}_{n}}$.
  • Trong $T$ có chứa số $-2$, đặt số lượng dãy có tính chất này là ${{c}_{n}}$.

Từ đó, ta dễ dàng chứng minh được hệ thức truy hồi

$ {{a}_{n+1}}=2{{a}_{n}} $

${{b}_{n+1}}={{a}_{n}}+2{{b}_{n}}+{{c}_{n}} $

$ {{c}_{n+1}}={{a}_{n}}+{{b}_{n}}+2{{c}_{n}} $

Chú ý rằng ${{a}_{n}}={{2}^{n}}$ và ${{a}_{n}}+{{b}_{n}}+{{c}_{n}}={{s}_{n}}$. Cộng hai công thức cuối lại, ta có

$${{b}_{n+1}}+{{c}_{n+1}}=2{{a}_{n}}+3({{b}_{n}}+{{c}_{n}})\Leftrightarrow {{s}_{n+1}}-{{2}^{n+1}}={{2}^{n+1}}+3({{s}_{n}}-{{2}^{n}})$$ hay

$${{s}_{n+1}}=3{{s}_{n}}+{{2}^{n}}\Leftrightarrow {{s}_{n+1}}+{{2}^{n+1}}=3({{s}_{n}}+{{2}^{n}}).$$

Với $n=1$, ta có $4$ dãy là $(1,1),(-1,-1),(-1,1),(1,-1)$ nên ${{s}_{1}}=4.$

Từ đẳng thức trên, ta có ${{s}_{n}}+{{2}^{n}}={{3}^{n-1}}({{s}_{1}}+{{2}^{1}})=2\cdot {{3}^{n}}$ nên ${{s}_{n}}=2\cdot {{3}^{n}}-{{2}^{n}}$.

Nhận xét: Bài toán thoạt nhìn có vẻ quen thuộc nhưng thật không đơn giản. Điều kiện đề cho là giá trị tuyệt đối của tất cả các tổng con từ vị trí lẻ đến vị trí chẵn bất kỳ đều không vượt quá $2$ buộc ta phải có đánh giá thích hợp mới có thể truy hồi được.

2. Bài tập áp dụng

Bài 1 (TP.HCM 2018): Hỏi có bao nhiêu hoán vị $(a_1,\ a_2,\ \ldots ,\ a_{164})$ của $164$ số nguyên dương đầu tiên sao cho $a_i \ne i$ và $ a_i \equiv i\text{ }(\bmod 41)$ với mọi $i=1,2,\ldots ,164?$

Bài 2: (Bài toán phát kẹo) Cô giáo có $10$ loại kẹo (mỗi loại có nhiều viên) và cần phát cho $30$ học sinh của lớp (một em nhận không quá $1$ viên/loại), giả sử rằng các em này có học lực đôi một khác nhau. Hỏi cô giáo có bao nhiêu cách phát kẹo, biết rằng nếu học sinh $A$ giỏi hơn $B$ thì $B$ có kẹo gì là $A$ có kẹo đó (tính cả trường hợp không em nào nhận được kẹo)?

Bài 3: (Bài toán con nhện) Một con nhện có $8$ cái chân, $8$ cặp vớ – giày khác nhau (vớ chỉ dùng chung với chiếc giày tương ứng). Con nhện có bao nhiêu thứ tự mang vớ và giày để sao cho trên cùng một chân, giày phải được mang vào sau vớ?